Sunteți pe pagina 1din 103

The possibility of the auditor's failure to recognize a misstatement

in an amount or a deviation from a prescribed control arises from

a. Statistical risk.
b. Sampling risk.
c. The standard error of the mean.
d. Nonsampling risk.
d. Nonsampling risk.

Which of the following best illustrates the concept of sampling


risk?

a. A randomly chosen sample may not be representative of the


population as a whole on the characteristic of interest.
b. An auditor may select audit procedures that are not appropriate
to achieve the specific objective.
c. An auditor may fail to recognize misstatements in the
documents examined for the chosen sample.
d. The documents related to the chosen sample may not be
available for inspection.
a. A randomly chosen sample may not be representative of the
population as a whole on the characteristic of interest.

During the audit of inventories an internal auditor specified a


precision of five percent instead of the four percent contained in
the preliminary audit program. What would be the impact of the
change in precision?

a. A decrease in population standard deviation.


b. An increase in population standard deviation.
c. A decrease in required sample size.
d. An increase in required sample size.
c. A decrease in required sample size.
An advantage of using statistical sampling techniques is that such
techniques

a. Mathematically measure risk.


b. Eliminate the need for judgmental decisions.
c. Define the values of precision and reliability required to provide
audit satisfaction.
d. Have been established in the courts to be superior to
judgmental sampling.
a. Mathematically measure risk.

As a result of sampling procedures applied as test of controls, an


auditor incorrectly assesses control risk lower than appropriate.
The most likely explanation for this situation is that

a. The deviation rates of both the auditor's sample and the


population exceed the tolerable rate.
b. The deviation rates of both the auditor's sample and the
population is less than the tolerable rate.
c. The deviation rate in the auditor's sample is less than the
tolerable rate, but the deviation rate in the population exceeds the
tolerable rate.
d. The deviation rate in the auditor's sample exceeds the tolerable
rate, but the deviation rate in the population is less than the
tolerable rate.
c. The deviation rate in the auditor's sample is less than the
tolerable rate, but the deviation rate in the population exceeds the
tolerable rate.

If certain forms are not consecutively numbered

a. Selection of a random sample probably is not possible.


b. Systematic sampling may be appropriate.
c. Stratified sampling should be used.
d. Random number tales can not be used.
b. Systematic sampling may be appropriate.

When performing a test of controls with respect to control over


cash disbursements, a CPA may use a systematic sampling
technique with a start at any randomly selected item. The biggest
disadvantage of this type of sampling is that the items in the
population

a. Must be recorded in a systematic pattern before the sample


can be drawn.
b. May occur in a systematic pattern, thus destroying the sample
randomness.
c. May systematically occur more than once in the sample.
d. Must be systematically replaced in the population after
sampling.
b. May occur in a systematic pattern, thus destroying the sample
randomness.

Given random selection, the same sample size, and same


tolerable rate for the testing of two unequal populations, the risk of
assessing control risk too low for the smaller population is

a. Higher than the risk of assessing control risk too low for the
larger population.
b. Lower than the risk of assessing control risk too low for the
larger population.
c. The same as the risk of assessing control risk too low for the
larger population.
d. Indeterminable relative to the risk of assessing control risk too
low for the larger population.
b. Lower than the risk of assessing control risk too low for the
larger population.
As lower acceptable levels of both audit risk and materiality are
established, the auditor should plan more work on individual
accounts to

a. Find smaller misstatements.


b. Find larger misstatements.
c. Increase the tolerable misstatement in the accounts.
d. Decrease the risk of assessing control risk too low.
a. Find smaller misstatements.

If the auditor is concerned that a population may contain critical


deviations, the determination of a sample size sufficient to include
at least one such deviation is a characteristic of

a. Discovery sampling.
b. Variables sampling.
c. Random sampling.
d. Dollar-unit sampling
a. Discovery sampling.

If all other factors specified in an attribute sampling plan remain


constant, changing the tolerable rate from 6% to 10%, and
changing the risk of assessing control risk too low from 3% to 7%
would cause the required sample size to

a. Increase.
b. Remain the same.
c. Decrease.
d. Change by 4%.
c. Decrease

The application of statistical sampling techniques is least related


to which of the following generally accepted auditing standards?
a. The work is to be adequately planned and assistants, if any,
are to be properly supervised.
b. In all matters relating to the assignment, an independence in
mental attitude is to be maintained by the auditor or auditors.
c. A sufficient understanding of internal control is to be obtained to
plan the audit and to determine the nature, timing, and extent of
the tests to be performed.
d. Sufficient competent evidential matter is to be obtained through
inspection, observation, inquiries and confirmations to afford a
reasonable basis for an opinion regarding the financial statements
under audit.
b. In all matters relating to the assignment, an independence in
mental attitude is to be maintained by the auditor or auditors.

An auditor plans to examine a sample of 20 checks for


countersignatures as prescribed by the client's controls. One of
the checks in the chosen sample of 20 cannot be found. The
auditor should consider the reasons for this limitation and

a. Evaluate the results as if the sample size had been 19.


b. Treat the missing check as a deviation for the purpose of
evaluating the sample.
c. Treat the missing check in the same manner as the majority of
the other 19 checks, i.e., countersigned or not.
d. Choose another check to replace the missing check in the
sample.
b. Treat the missing check as a deviation for the purpose of
evaluating the sample.

If the achieved precision range of a statistical sample at a given


reliability level is greater than the desired range, this is an
indication that the

a. Standard deviation was larger than expected.


b. Standard deviation was less than expected.
c. Population was larger than expected.
d. Population was smaller than expected.
a. Standard deviation was larger than expected.

An auditor examining inventory may appropriately apply sampling


for attributes in order to estimate the

a. Average price of inventory items.


b. Percentage of slow-moving inventory items.
c. Dollar value of inventory.
d. Physical quantity of inventory items.
b. Percentage of slow-moving inventory items.

In attribute sampling, a 10% change in which of the following


factors normally will have the least effect on the size of a
statistical sample?

a. Population size.
b. Precision (confidence interval).
c. Reliability (confidence level).
d. Standard deviation.
a. Population size.

An auditor initially planned to use unrestricted random sampling


with replacement in the examination of accounts receivable.
Later, the auditor decided to use unrestricted random sampling
without replacement. As a result only of this decision, the sample
size should

a. Increase.
b. Remain the same.
c. Decrease.
d. Be recalculated using a binomial distribution.
c. Decrease
If the size of the sample to be used in a particular attribute test
has not been determined by utilizing statistical concepts, but the
sample has been chosen in accordance with random selection
procedures,

a. No inferences can be drawn from the sample.


b. The auditor has committed a nonsampling error.
c. The auditor may or may not achieve desired precision at the
desired level of confidence.
d. The auditor will have to evaluate the results by reference to the
principles of discovery sampling.
c. The auditor may or may not achieve desired precision at the
desired level of confidence.

Jones, CPA, believes the industry-wide occurrence rate of client


billing deviations is 3% and has established a maximum
acceptable occurrence rate of 5%. In the review of client invoices
Jones should use
a. Discovery sampling.
b. Attribute sampling.
c. Stratified sampling.
d. Variable sampling.
b. Attribute sampling.

The major reason that the difference and ratio estimation methods
would be expected to produce audit efficiency is that the

a. Number of members of the population of differences or ratios is


smaller than the number of members of the population of book
values.
b. Beta risk may be completely ignored.
c. Calculations required in using difference or ratio estimation are
less arduous and fewer than those required when using direct
estimation.
d. Variability of the populations of differences or ratios is less than
that of the populations of book values or audited values.
d. Variability of the populations of differences or ratios is less than
that of the populations of book values or audited values.

In estimation sampling for variables, which of the following must


be known in order to estimate the appropriate sample size
required to meet the auditor's needs in a given situation?

a. The total amount of the population.


b. The desired standard deviation.
c. The desired confidence level.
d. The estimated rate of misstatement in the population.
c. The desired confidence level.

When using a statistical sampling plan, the auditor would probably


require a smaller sample if the

a. Population increases.
b. Desired precision interval narrows.
c. Desired reliability decreases.
d. Expected misstatement occurrence rate increases.
c. Desired reliability decreases.

In performing tests of controls, the auditor will normally find that

a. The level of risk is directly proportionate to the rate of


misstatement.
b. The rate of deviations in the sample exceeds the rate of
misstatement in the accounting records.
c. The rate of misstatement in the sample exceeds the rate of
deviations.
d. All unexamined items result in misstatements in the accounting
records.
b. The rate of deviations in the sample exceeds the rate of
misstatement in the accounting records.

An underlying feature of random-based selection of items is that


each

a. Stratum of the accounting population be given equal


representation in the sample.
b. Item in the accounting population be randomly ordered.
c. Item in the accounting population should have an opportunity to
be selected.
d. Item must be systematically selected using replacement.
c. Item in the accounting population should have an opportunity to
be selected.

When planning a sample for a substantive test of details, an


auditor should consider tolerable misstatement for the test. This
consideration should

a. Be related to the auditor's business risk.


b. Not be adjusted for qualitative factors.
c. Be related to preliminary judgments about materiality levels.
d. Not be changed during the audit process.
c. Be related to preliminary judgments about materiality levels.

In assessing sampling risk, the risk of incorrect rejection and the


risk of assessing control risk too high relate to the

a. Efficiency of the audit.


b. Effectiveness of the audit.
c. Selection of the sample.
d. Audit quality controls.
a. Efficiency of the audit.
Using statistical sampling to assist in verifying the year-end
accounts payable balance, an auditor has accumulated the
following data:

Balance determined
Accounts Balance by the auditor
Population 4,100 $5,000,000 ?
Sample 200 $ 250,000 $300,000

Using the ratio estimation technique, the auditor's estimate of


year-end accounts payable balance would be

a. $6,150,000.
b. $6,000,000.
c. $5,125,000.
d. $5,050,000.
b. $6,000,000.

An important statistic to consider when using a statistical


sampling audit plan is the population variability. The population
variability is measured by the

a. Sample mean.
b. Standard deviation.
c. Standard error of the sample mean.
d. Estimated population total minus the actual population total.
b. Standard deviation.

In variables estimation sampling, the sample standard deviation is


used to calculate the

a. Point estimate of central tendency.


b. Tainting of the sample interval.
c. Risk of incorrect acceptance.
d. Adjusted allowance for sampling risk.
d. Adjusted allowance for sampling risk.

Use of the ratio estimation sampling technique to estimated dollar


amounts is in appropriate when:

a. The total book value is known and corresponds to the sum of


all the individual book values.
b. A book value for each sample item is unknown.
c. There are some observed differences between audited values
and book values.
d. The audited values are nearly proportional to the book value.
b. A book value for each sample item is unknown.

After partially completing an internal control review of the


accounts payable department, an auditor suspects that some type
of fraud has occurred. To ascertain whether the fraud is present,
the best sampling approach would be to use

a. Simple random sampling to select a sample of vouchers


processed by the department during the past year.
b. Probability-proportional-to-size sampling to select a sample of
vouchers processed by the department during the past year.
c. Discovery sampling to select a sample of vouchers processed
by the department during the past year.
d. Judgmental sampling to select a sample of vouchers processed
by clerks identified by the department manager as acting
suspiciously.
c. Discovery sampling to select a sample of vouchers processed
by the department during the past year.

Management is legally required to prepare a shipping document


for all movement of hazardous materials. The document must be
filed with bills of lading. Management expects 100 percent
compliance with the procedure. Which of the following sampling
approaches would be most appropriate?
a. Attributes sampling.
b. Discovery sampling.
c. Targeted sampling.
d. Variables sampling.
b. Discovery sampling.

Which of the following must be known to evaluate the results of


an attribute sample?

a. Estimated dollar value of the population.


b. Standard deviation of the sample values.
c. Actual size of the sample selected.
d. Finite population correction factor.
c. Actual size of the sample selected.

If all other factors specified in an attribute sampling plan remain


constant, decreasing the confidence level from 95 percent to 90
percent would cause the required sample size to

a. Increase.
b. Decrease.
c. Change by 5 percent.
d. Remain the same.
b. Decrease

Internal auditors employ confidence levels in the context of audit


sampling. In a given sample plan the confidence level:

a. Is a decision variable which the internal auditor specifies after


considering the economic consequences of drawing the wrong
conclusion as a result of sampling error.
b. Is a characteristic of the audit population and is not under the
direct control of the auditor.
c. Is essentially a measure of the accuracy of the sample results
obtained after the sample has been selected and tested.
d. Is not normally specified before the sample size is determined.
Rather it is computed once the sample has been selected and
tested.
a. Is a decision variable which the internal auditor specifies after
considering the economic consequences of drawing the wrong
conclusion as a result of sampling error.

An advantage of statistical sampling over nonstatistical sampling


is that statistical sampling helps an auditor to

a. Eliminate the risk of nonsampling errors.


b. Reduce the level of audit risk and materiality to a relatively low
amount.
c. Measure the sufficiency of the evidential matter obtained.
d. Minimize the failure to detect errors and fraud.
c. Measure the sufficiency of the evidential matter obtained.

An auditor tested a population by examining 60 items selected


judgmentally and found one error. The main limitation of the
auditor's sample is the inability to:

a. Quantify sampling risk.


b. Quantify the acceptable error rate.
c. Project the acceptable error rate.
d. Determine whether the sample is random
a. Quantify sampling risk.

An auditor used a mean-per-unit sampling plan to estimate the


average cost of repairing photocopy machines. The sample size
was 50 and population size was 2,000. The mean of the sample
was $75. The standard deviation was $14 and the standard error
of the mean was $2. What is the confidence interval at a 95%
confidence level (Z = 2)?
a. $94,000 to $206,000.
b. $142,000 to $158,000.
c. $122,000 to $178,000.
d. $146,000 to $150,000.
b. $142,000 to $158,000.

What effect does an increase in the standard deviation have on


the required sample size of mean-per-unit estimation and
probability proportion to size sampling? Assume no change in any
of the other characteristics of the population and no change in
desired precision and confidence.

Mean-per-unit Probability Propor-


Estimation tional to Size
a. Increase in sample size. Increase in sample size.
b. No change in sample size. Decrease in sample size.
c. Increase in sample size. No change in sample size.
d. Decrease in sample size. No change in sample size.
c. Increase in sample size. No change in sample size.

An internal auditor wants to select a statistically representative


sample from a population of 475 inventory control sheets. Each
sheet lists the description, physical count, bar code, and unit cost
for 50 inventory items. The auditor uses a random number table
to construct the sample; the first two columns are listed below.
14326 is the randomly chosen starting point; the sample's first
item is found on page 143, line 26. (The route used by the auditor
is down Column A to the top of Column B.)
Column A Column B
75233 06852
14326 42904
76562 64854
28123 04978
64227 33150
80938 04301
22539 41240
29452 69521
Where is the fifth item in the sample located?

a. Page 809, line 38.


b. Page 429, line 04.
c. Page 331, line 50.
d. Page 068, line 52.
c. Page 331, line 50.

Which of the following is an advantage of systematic sampling


over random number sampling?

a. It provides a stronger basis for statistical conclusions.


b. It enables the auditor to use the more efficient "sampling with
replacement" tables.
c. There may be correlation between the location of items in the
population, the feature of sampling interest, and the sampling
interval.
d. It does not require establishment of correspondence between
random numbers and items in the population.
d. It does not require establishment of correspondence between
random numbers and items in the population.

Statistical sampling generally may be applied to test internal


control when the client's internal control procedures:

a. Depend primarily on appropriate segregation of duties.


b. Are carefully reduced to writing and are included in client
accounting manuals.
c. Leave an audit trail in the form of evidence of performance.
d. Enable the detection of material fraud in the accounting
records.
c. Leave an audit trail in the form of evidence of performance.
To determine sample size in an attribute sampling application,
what must be specified?

a. Population mean, expected deviation rate, allowance for


sampling risk.
b. Allowance for sampling risk, risk of assessing control risk too
low, standard deviation.
c. Allowance for sampling risk, risk of assessing control risk too
low, expected deviation rate.
d. Population mean, standard deviation, allowance for sampling
risk.
c. Allowance for sampling risk, risk of assessing control risk too
low, expected deviation rate.

The objective of the tolerable rate in sampling for tests of controls


is to:

a. Determine the probability of the auditor's conclusion based


upon reliance factors.
b. Determine that financial statements taken as a whole are not
materially in error.
c. Estimate the reliability of substantive tests.
d. Estimate the range of procedural deviations in the population.
d. Estimate the range of procedural deviations in the population.

In testing accounts receivable, an auditor sends out positive


confirmation requests to 100 randomly selected customers. A
customer returns the confirmation indicating that the balance is
correct when, in fact, the balance is overstated. This is an
example of:

a. Projected misstatement.
b. Sampling error.
c. Standard error.
d. Nonsampling error.
d. Nonsampling error.

Which of the following factors does an auditor need to consider in


planning a particular audit sample for a test of control?

a. Number of items in the population.


b. Total dollar amount of the items to be sampled.
c. Acceptable level of risk of assessing control risk too low.
d. Tolerable misstatement.
c. Acceptable level of risk of assessing control risk too low.

When using a statistical sampling plan, the auditors would


probably require a smaller sample if the:

a. Population increases.
b. Desired allowance for sampling risk decreases.
c. Desired risk of incorrect acceptance increases.
d. Expected deviation rate increases.
c. Desired risk of incorrect acceptance increases.

A number of factors influences the sample size for a substantive


test of details of an account balance. All other factors being equal,
which of the following would lead to a larger sample size?

a. Lower assessment of control risk.


b. Greater reliance on analytical procedures.
c. Smaller expected frequency of misstatements.
d. Smaller measure of tolerable misstatement.
d. Smaller measure of tolerable misstatement.

Mr. Shoe, CPA, is planning substantive tests of additions to


property. There are 75 additions and he plans to vouch all those
over $10,000, of which there are 15, and apply analytical
procedures to the remaining balance. The audit approach may be
described most precisely as
a. Audit sampling.
b. Nonstatistical sampling.
c. Judgment sampling.
d. None of the above.
d. None of the above.

Mr. Shoe, CPA is planning tests of controls of cash


disbursements. There are 3,000 disbursements and he plans to
vouch 60 picked haphazardly from the cash disbursements
journal, for the period January 1 to September 30, 20X7. This
audit approach may be described most precisely as

a. Audit sampling.
b. Statistical sampling.
c. Nonstatistical sampling.
d. None of the above.
c. Nonstatistical sampling.

Mr. Shoe, CPA, is planning confirmation of accounts receivable.


There are 500 customer balances and, based on the condition of
the accounting records and his past experience with the client, he
plans to send 50 confirmation requests to customers selected,
using a random number table, from the aged trial balance of
accounts receivable. Mr. Shoe plans to evaluate confirmation
responses qualitatively and by multiplying the average
misstatement in the 50 responses by 500. This audit approach
may be described most precisely as

a. Audit sampling.
b. Statistical sampling.
c. Nonstatistical sampling.
d. None of the above.
c. Nonstatistical sampling.
Whenever an auditor uses audit sampling all of the following
requirements apply except

a. Projection of the misstatement results of the sample to


population from which the sample was selected.
b. Calculation of the appropriate sample size using one of several
formulas appropriate for audit use.
c. Selection of sample items in such a way that the sample can be
expected to be representative of the population.
d. Consideration of the relationship of the sample to relevant audit
objectives
b. Calculation of the appropriate sample size using one of several
formulas appropriate for audit use.

An attribute sampling plan that is one type of proportional


sampling is

a. Stop-or-go sampling.
b. Stratified mean per unit estimation.
c. Ratio estimation.
d. Probability-proportional-to-size sampling.
d. Probability-proportional-to-size sampling.

SAS No. 39 on audit sampling was issued by the AICPA in

a. 1981.
b. 1963.
c. 1972.
d. None of the above.
a. 1981.

One of the most significant features of SAS No. 39 is that it, for
the first time
a. Required statistical sampling to comply with generally accepted
auditing standards in specified circumstances.
b. Equated statistical and nonstatistical sampling in a common
approach.
c. Made clear that in determining the extent of a particular audit
test an auditor could consider using statistical sampling.
d. Explained the relationship of common statistical terms to audit
concepts such as materiality and audit risk.
b. Equated statistical and nonstatistical sampling in a common
approach.

Which of the following errors may be correctly described as


sampling error?

a. Failure to define the population properly.


b. Failure to obtain a random start.
c. Failure to recognize a misstatement that exists in the sample.
d. None of the above.
d. None of the above.

Viewed from a statistical sampling perspective, the generally


accepted auditing standards that take on a special meaning are

a. All fieldwork standards and the first general standard.


b. All reporting standards and the third general standard.
c. All general standards and the third standard of fieldwork.
d. None of the above.
a. All fieldwork standards and the first general standard.

The following five questions (10 to 14) are concerned with the
following description of a sampling application: The auditor is
planning the confirmation of accounts receivable at 12/31/X7.
Accounts receivable are maintained in a tub file of unpaid sales
invoices filed by date. When a customer remits cash it is matched
to the oldest invoices first. Invoice numbers are preprinted on
remittance advices (tear-off portion of invoice mailed to the
customer). In defining population, frame and sample unit, the
auditor would most logically in the circumstances

Define the total accounts receivable as of 12/31/X7 as the

a. Frame.
b. Population.
c. Sample unit.
d. None of the above
b. Population.

The following five questions (10 to 14) are concerned with the
following description of a sampling application: The auditor is
planning the confirmation of accounts receivable at 12/31/X7.
Accounts receivable are maintained in a tub file of unpaid sales
invoices filed by date. When a customer remits cash it is matched
to the oldest invoices first. Invoice numbers are preprinted on
remittance advices (tear-off portion of invoice mailed to the
customer). In defining population, frame and sample unit, the
auditor would most logically in the circumstances

Define a sales invoice unpaid at 12/31/X7 as the

a. Frame.
b. Population.
c. Sample unit.
d. None of the above.
c. Sample unit.

The following five questions (10 to 14) are concerned with the
following description of a sampling application: The auditor is
planning the confirmation of accounts receivable at 12/31/X7.
Accounts receivable are maintained in a tub file of unpaid sales
invoices filed by date. When a customer remits cash it is matched
to the oldest invoices first. Invoice numbers are preprinted on
remittance advices (tear-off portion of invoice mailed to the
customer). In defining population, frame and sample unit, the
auditor would most logically in the circumstances

Define the preprinted invoice number on a remittance advice as


the

a. Frame.
b. Population.
c. Sample unit.
d. None of the above.
d. None of the above.

The following five questions (10 to 14) are concerned with the
following description of a sampling application: The auditor is
planning the confirmation of accounts receivable at 12/31/X7.
Accounts receivable are maintained in a tub file of unpaid sales
invoices filed by date. When a customer remits cash it is matched
to the oldest invoices first. Invoice numbers are preprinted on
remittance advices (tear-off portion of invoice mailed to the
customer). In defining population, frame and sample unit, the
auditor would most logically in the circumstances

Define the tub file of unpaid sales invoices as the

a. Frame.
b. Population.
c. Sample unit.
d. None of the above.
a. Frame.

The following five questions (10 to 14) are concerned with the
following description of a sampling application: The auditor is
planning the confirmation of accounts receivable at 12/31/X7.
Accounts receivable are maintained in a tub file of unpaid sales
invoices filed by date. When a customer remits cash it is matched
to the oldest invoices first. Invoice numbers are preprinted on
remittance advices (tear-off portion of invoice mailed to the
customer). In defining population, frame and sample unit, the
auditor would most logically in the circumstances

Define a balance due from an individual customer at 12/31/X7 as


the

a. Frame.
b. Population.
c. Sample unit.
d. None of the above.
d. None of the above.

An auditor is selecting a random sample of cash disbursements


for a test of controls and finds that one of the checks that
corresponds to a selected random number is a voided check. The
auditor should

a. Reduce the sample size by one.


b. Treat the check as a deviation.
c. Select the check immediately before or after the one that
corresponds to the random number.
d. None of the above.
a. Reduce the sample size by one.

An auditor is using the probability-proportional-to-size selection


method. Using this method the chance of selecting a $100 item
compared to selecting a $500 item is

a. 20%.
b. 50%.
c. Greater but not determinable without knowing the dollar amount
of the population.
d. Less but not determinable without knowing the dollar amount of
the population.
a. 20%.

An auditor selecting a nonstatistical sample may use any of the


following methods except

a. Stratified selection.
b. Haphazard selection.
c. Judgmental selection.
d. Simple random selection.
c. Judgmental selection.

In tests of controls, the upper limit on deviations that would


support the auditor's planned assessed level of control risk is
referred to as

a. Upper limit rate.


b. Maximum rate.
c. Expected rate.
d. Tolerable rate.
d. Tolerable rate.

In tests of controls using statistical sampling, one minus reliability


is generally the equivalent to the

a. Risk of assessing control risk too high.


b. Risk of assessing control risk too low.
c. Confidence level.
d. Upper precision limit.
b. Risk of assessing control risk too low.

In accordance with professional standards the risk of assessing


control risk too low should generally be
a. 5% or 10%.
b. Inversely related to the planned assessed level of control risk.
c. Never greater than 20%.
d. Related to audit efficiency
a. 5% or 10%.

When acceptable UPL is less than achieved UPL, the auditor may
do all of the following except

a. Reconsider the definition of a deviation.


b. Increase sample size of the test of controls.
c. Increase sample size of the related substantive test.
d. Reconsider the specified tolerable rate
d. Reconsider the specified tolerable rate

In using the tables for fixed-sample-size attribute sampling in the


text, sample size is determined by finding the intersection of the
expected rate and

a. Risk of assessing control risk too low.


b. Population size.
c. Tolerable rate.
d. Reliability level.
c. Tolerable rate.

In using the text tables for fixed-sample size attribute sampling for
evaluation of sample results, the upper precision limit is found by
reading up from the number of deviations that is in the row for the
relevant

a. Risk of assessing control risk too low.


b. Sample size.
c. Tolerable rate.
d. Reliability level.
b. Sample size.

Risk Factors for Assessing Control Risk Too Low


Number of
Deviations 10% 5%
0 2.4 3.0
1 3.9 4.8
2 5.4 6.3
3 6.7 7.8

If an auditor planning a test of controls sample specifies a risk of


assessing control risk too low of 5% and a tolerable rate of 6%,
and expects no deviations, the planned sample size should be

a. 50.
b. 60.
c. 80.
d. 18.
a. 50.

Risk Factors for Assessing Control Risk Too Low


Number of
Deviations 10% 5%
0 2.4 3.0
1 3.9 4.8
2 5.4 6.3
3 6.7 7.8

If an auditor planning a test of controls sample specifies a risk of


assessing control risk too low of 5% and a tolerable rate of 6%
and expects 1 deviation, the planned sample size should be

a. 50.
b. 60.
c. 80.
d. 29.
b. 60

Risk Factors for Assessing Control Risk Too Low


Number of
Deviations 10% 5%
0 2.4 3.0
1 3.9 4.8
2 5.4 6.3
3 6.7 7.8

If an auditor is evaluating a test of controls sample of 50 items


and specifies a risk of assessing control risk too low of 10% and
finds 2 deviations, the approximate achieved UPL is

a. 6%.
b. 10%.
c. 4%.
d. 11%.
d. 11%

Risk Factors for Assessing Control Risk Too Low


Number of
Deviations 10% 5%
0 2.4 3.0
1 3.9 4.8
2 5.4 6.3
3 6.7 7.8

If an auditor is evaluating a test of controls sample of 50 items


and specifies a risk of assessing control risk too low of 5% and
finds 1 deviation, the approximate achieved UPL is

a. 6%.
b. 10%.
c. 4%.
d. 11%.
b. 10%

Risk Factors for Assessing Control Risk Too Low


Number of
Deviations 10% 5%
0 2.4 3.0
1 3.9 4.8
2 5.4 6.3
3 6.7 7.8

An auditor selects a test of controls samples of 50. The tolerable


rate was specified as 6%. The auditor finds one deviation and
wishes to evaluate the sample at a 10% risk of assessing control
risk too low. The sample results

a. Support the planned level of control risk.


b. Do not support the planned level of control risk.
c. Are indeterminate without more data.
d. None of the above.
b. Do not support the planned level of control risk.

If a population has a standard deviation of 300, the standard error


of the mean for sample sizes of 100 with a mean of 5 is

a. 30.
b. 20.
c. 3.
d. Indeterminate without more data.
a. 30.

Accounting estimation using unstratified mean per unit estimation


would be used when
a. An account balance is to be determined by statistical sampling
and the values of individual items comprising the balance are not
known.
b. An account balance is to be evaluated as materially misstated
or not materially misstated and the values of individual items
comprising the balance are known.
c. An account balance is to be determined by statistical sampling
and the values of individual items comprising the balance are
known.
d. An account balance is to be evaluated as materially misstated
or not materially misstated and the number of items comprising
the account balance is not known.
a. An account balance is to be determined by statistical sampling
and the values of individual items comprising the balance are not
known.

The susceptibility of an account balance to material misstatement


given its own characteristics and environmental characteristics is
referred to as

a. Audit risk.
b. Inherent risk.
c. Control risk.
d. Detection risk
b. Inherent risk.

The beta risk associated with a substantive test applied using


sampling is an aspect of

a. Audit risk.
b. Inherent risk.
c. Control risk.
d. Detection risk
d. Detection risk
In judgmentally determining the control risk (CR), the auditor
should never allow control risk to be

a. 100%.
b. 20%.
c. 50%.
d. 5%.
d. 5%.

In judgmentally determining the other procedures risk (AP), the


auditor should rarely allow AP to be less than

a. 100%.
b. 20%.
c. 50%.
d. 5%.
c. 50%.

In using the audit risk model, the auditor generally sets audit risk
at

a. 0 to 100%.
b. 50% to 100%.
c. 5% to 10%.
d. 20% to 80%.
c. 5% to 10%.

If an auditor has established audit risk as 10%, control and other


procedures risk each as 50%, and inherent risk at the maximum,
the desired beta risk is

a. 20%.
b. 40%.
c. 80%.
d. 100%.
b. 40%.

If an auditor has established audit risk as 5%, control risk and


other procedures risk each as 50%, and inherent risk at the
maximum, the desired beta risk is

a. 20%.
b. 40%.
c. 80%.
d. 100%.
a. 20%

Beta risk is incorporated in the formula for estimation of sample


size using variable sampling by adjustment of

a. Reliability.
b. Standard deviation.
c. Confidence coefficient.
d. Precision
d. Precision

An auditor is using variable sampling for a substantive test. The


estimated audited value is $90,000, the book value is $100,000
achieved precision is $10,000, and the standard error of the mean
is 2. The decision interval is

a. $80,000 to $100,000.
b. $90,000 to $110,000.
c. $85,000 to $95,000.
d. $95,000 to $105,000.
b. $90,000 to $110,000.

Alpha risk is incorporated in the formula for estimation of the


sample size using variable sampling by adjustment of

a. Materiality.
b. Standard deviation.
c. Confidence coefficient.
d. Precision
c. Confidence coefficient.

An auditor is planning the confirmation of accounts receivable.


The total of debit balances in the aged trial balance of receivables
is $2,000,000. The auditor has decided that the tolerable
misstatement (basic precision) for this sampling application is
$50,000 and that a risk of incorrect acceptance of 5% is
appropriate. There are 2500 customer balances. The auditor has
available the following portion of a table of Poisson factors. The
auditor decides to use PPS sampling.

Confidence Levels Number of Occurrences 90% 95%


0 2.4 3.0
1 3.9 4.8
2 5.4 6.3
3 6.7 7.8

The auditor's estimated sample size is approximately

a. 80.
b. 120.
c. 160.
d. 200
b. 120

An auditor is planning the confirmation of accounts receivable.


The total of debit balances in the aged trial balance of receivables
is $2,000,000. The auditor has decided that the tolerable
misstatement (basic precision) for this sampling application is
$50,000 and that a risk of incorrect acceptance of 5% is
appropriate. There are 2500 customer balances. The auditor has
available the following portion of a table of Poisson factors. The
auditor decides to use PPS sampling.

Confidence Levels Number of Occurrences 90% 95%


0 2.4 3.0
1 3.9 4.8
2 5.4 6.3
3 6.7 7.8

The sampling interval using the PPS systematic selection method


is approximately

a. 50,000.
b. 6.
c. 16,700.
d. 240.
c. 16,700

An auditor is planning the confirmation of accounts receivable.


The total of debit balances in the aged trial balance of receivables
is $2,000,000. The auditor has decided that the tolerable
misstatement (basic precision) for this sampling application is
$50,000 and that a risk of incorrect acceptance of 5% is
appropriate. There are 2500 customer balances. The auditor has
available the following portion of a table of Poisson factors. The
auditor decides to use PPS sampling.

Confidence Levels Number of Occurrences 90% 95%


0 2.4 3.0
1 3.9 4.8
2 5.4 6.3
3 6.7 7.8

In evaluating the results of this sample, the basic bound will be


a. 100,000.
b. 150,000.
c. 25,000.
d. 50,000
d. 50,000

An auditor has selected sample size of 100 with a specified risk of


incorrect acceptance of 5%. The account balance being sampled
is $100,000 and the auditor has found the following two
misstatements in examining sample items.

Book Value Audited Value


2,000 1,500
1,000 500

Confidence Levels
Number of Occurrences 90% 95%
0 2.4 3.0
1 3.9 4.8
2 5.4 6.3
3 6.7 7.8

The basic bound in this sampling application is

a. 1,000.
b. 3,000.
c. 6,000.
d. 10,000.
b. 3,000

An auditor has selected sample size of 100 with a specified risk of


incorrect acceptance of 5%. The account balance being sampled
is $100,000 and the auditor has found the following two
misstatements in examining sample items.
Book Value Audited Value
2,000 1,500
1,000 500

Confidence Levels
Number of Occurrences 90% 95%
0 2.4 3.0
1 3.9 4.8
2 5.4 6.3
3 6.7 7.8

The first addition to the basic bound for finding one misstatement
is

a. 500.
b. 900.
c. 1200.
d. 300
b. 900

An auditor has selected sample size of 100 with a specified risk of


incorrect acceptance of 5%. The account balance being sampled
is $100,000 and the auditor has found the following two
misstatements in examining sample items.

Book Value Audited Value


2,000 1,500
1,000 500

Confidence Levels
Number of Occurrences 90% 95%
0 2.4 3.0
1 3.9 4.8
2 5.4 6.3
3 6.7 7.8
The second addition to the basic bound for finding a second
misstatement is

a. 1200.
b. 900.
c. 450.
d. 375.
d. 375

An auditor has selected sample size of 100 with a specified risk of


incorrect acceptance of 5%. The account balance being sampled
is $100,000 and the auditor has found the following two
misstatements in examining sample items.

Book Value Audited Value


2,000 1,500
1,000 500

Confidence Levels
Number of Occurrences 90% 95%
0 2.4 3.0
1 3.9 4.8
2 5.4 6.3
3 6.7 7.8

In planning this sample the tolerable misstatement used in


computing sample size was

a. 10,000.
b. 6,000.
c. 3,000.
d. 2,500
c. 3,000
An auditor has selected sample size of 100 with a specified risk of
incorrect acceptance of 5%. The account balance being sampled
is $100,000 and the auditor has found the following two
misstatements in examining sample items.

Book Value Audited Value


2,000 1,500
1,000 500

Confidence Levels
Number of Occurrences 90% 95%
0 2.4 3.0
1 3.9 4.8
2 5.4 6.3
3 6.7 7.8

If the auditor believed that a misstatement of $4,000 would


materially misstate the account balance, then

a. The account balance should be rejected.


b. The account balance should be accepted.
c. The sample results do not permit a decision without additional
testing.
d. The results are indeterminate because the maximum
misstatement of overstatement is exactly equal to $4,000.
a. The account balance should be rejected.

An auditor has selected sample size of 100 with a specified risk of


incorrect acceptance of 5%. The account balance being sampled
is $100,000 and the auditor has found the following two
misstatements in examining sample items.

Book Value Audited Value


2,000 1,500
1,000 500
Confidence Levels
Number of Occurrences 90% 95%
0 2.4 3.0
1 3.9 4.8
2 5.4 6.3
3 6.7 7.8

The sample results indicate a most likely misstatement of


overstatement of

a. 0.
b. 100.
c. 500.
d. 750
d. 750

All of the following are advantages of PPS sampling except

a. Large items have a higher probability of selection.


b. It is not necessary to estimate the standard deviation of the
population.
c. Understated items have a lower probability of selection.
d. Several account balances can be combined and treated as one
population.
c. Understated items have a lower probability of selection.

For a sampling application to qualify as a statistical sample, which


of the following is a requirement:

a. Random number table must be used.


b. Sample size must be determined using a valid statistical
formula.
c. Judgment must not be allowed to influence evaluation of
sample results.
d. Sample results must be mathematically evaluated
d. Sample results must be mathematically evaluated

The primary advantage of statistical sampling is that it allows the


auditor to

a. Reach a conclusion on an account balance without examining


every item in the balance.
b. Reduce audit costs because sample sizes are smaller.
c. Measure the uncertainty associated with examining less than
every item in an account balance.
d. Reduce sampling error.
c. Measure the uncertainty associated with examining less than
every item in an account balance.

When using systematic selection an auditor needs to guard


against a biased sample. To avoid a biased sample the auditor
should do all of the following except:

a. Be satisfied that the population is in random order.


b. Be sure to use only one random start.
c. Continue sample selection until the population is exhausted.
d. Be sure to not substitute an adjacent unselected population
item for a sample item that cannot be located
b. Be sure to use only one random start

To evaluate whether all shipments are billed, the auditor should


use which of the following as the sampling frame:

a. Prenumbered sales invoice.


b. Accounts in the accounts receivable subsidiary ledger.
c. Prenumbered shipping documents.
d. Entries in the sales journal.
c. Prenumbered shipping documents.
Which of the following steps in a statistical test of controls is
generally different than in a nonstatistical test of the same type.

a. Define the audit objective.


b. Define the population relevant to the objective.
c. Determine the sample size.
d. Be alert for deviations not defined as testable attributes
c. Determine the sample size.

Attribute sampling with a fixed size requires the auditor to specify


all of the following to determine sample size except:

a. Tolerable rate.
b. Population size.
c. Risk of assessing control risk too low.
d. Expected deviation rate.
b. Population size.

When the results of an attribute sample indicate that the tolerable


rate is less than the achieved upper precision limit, the auditor
could appropriately do all of the following but:

a. Increase sample size.


b. Decide not to rely on the control procedures tested.
c. Revise the definition of a deviation.
d. Increase the acceptable upper precision limit.
d. Increase the acceptable upper precision limit.

The estimated value in an estimation sampling plan is also called:

a. Precision interval.
b. Proportional allocation.
c. Point estimate.
d. Population difference.
c. Point estimate.
The statistical concept of beta risk is identical with which of the
following concepts explained in SAS No. 39:

a. Risk of incorrect rejection.


b. Risk of incorrect acceptance.
c. Audit risk.
d. Substantive test risk.
b. Risk of incorrect acceptance.

If individual book values are not available for sample selection,


the most appropriate sampling approach is:

a. Difference estimation.
b. Unstratified mean per unit estimation.
c. Ratio estimation.
d. Stratified mean per unit estimation
b. Unstratified mean per unit estimation

If the auditor fails to recognize that an invoice selected in a


sample has been misfooted, this is an example of:

a. Sampling error.
b. Standard error of the mean.
c. Nonsampling error.
d. Intolerable misstatement.
c. Nonsampling error.

Tests of control can be made using a statistical sample. Which of


the following sampling methods can be used for testing controls:

a. Unstratified mean per unit estimation.


b. Probability proportional to size sampling.
c. Stratified mean per unit estimation.
d. Ratio estimation.
b. Probability proportional to size sampling.
In testing controls, the auditor's evaluation of sample results
would include a conclusion on whether:

a. Achieved monetary precision is not greater than planned


precision.
b. The class of transactions does not include misstatements in
excess of an amount material to the financial statements.
c. Procedural deviations in the class of transactions do not
exceed a specified rate.
d. Planned confidence level does not exceed achieved confidence
c. Procedural deviations in the class of transactions do not
exceed a specified rate.

Important aspects of an audit in accordance with professional


standards are proper planning, supervision, and review. These
measures help to reduce:

a. Sampling error.
b. Nonsampling error.
c. Tolerable misstatement.
d. Standard error of the mean
b. Nonsampling error.

Unexamined items are items included in sample selection that


cannot be located. In making tests of controls, the proper
approach to unexamined items is

a. Apply alternative procedures.


b. Substitute the item immediately before or after the selected
item.
c. Treat the item as a misstatement or deviation.
d. Reduce sample size and evaluate sample results based on
reduced sample size
c. Treat the item as a misstatement or deviation.
In making a nonstatistical sample, which of the following selection
methods is least appropriate for the auditor to use?

a. Systematic selection.
b. Block selection.
c. Stratified random selection.
d. Unstratified random selection.
b. Block selection.

In audit hypothesis testing, the decision interval is calculated by

a. One minus combined reliability.


b. Estimated audited value plus and minus precision.
c. Combined reliability minus beta risk.
d. Book value plus and minus precision.
d. Book value plus and minus precision.

Beta risk is controlled in classical hypothesis testing by

a. Varying reliability.
b. Adjusting the ratio of precision to materiality.
c. Rejecting book value in appropriate circumstances.
d. Varying the assessed level of control risk
b. Adjusting the ratio of precision to materiality.

The probability-proportional-to-size (PPS) sampling model


incorporates all of the following assumptions except

a. The population misstatement rate should be less than 10%.


b. The population should contain 2,000 or more items.
c. The account balances in the population should be similar in
size.
d. The amount of misstatement in an account balance cannot be
more than book value.
c. The account balances in the population should be similar in
size.

With respect to the probability of large recorded amounts being


selected, probability-proportional-to-size sampling is most similar
to

a. Systematic selection.
b. Random sampling without replacement.
c. Random systematic selection.
d. Stratified selection
d. Stratified selection

Which of the following statements is correct regarding the


auditor's determination of materiality:

a. The planning level of materiality should normally be the larger


of the amount considered for the balance sheet versus the
income statement.
b. The auditors' planning level of materiality may be
disaggregated into smaller "tolerable misstatements" for the
various accounts.
c. Auditors may use various rules of thumb to arrive at an
evaluation level of materiality, but not for determining the planning
level of materiality.
d. The amount used for the planning should equal that used for
evaluation.
b. The auditors' planning level of materiality may be
disaggregated into smaller "tolerable misstatements" for the
various accounts.

When using sampling for tests of controls, which of the following


audit consequences may follow?
a. If sample results indicate that the control is operating
effectively, but in fact it is not, control risk will be assessed too
low.
b. If sample results indicate that the control is operating
effectively, but in fact it is not, control risk will be assessed too
high.
c. If sample results indicate that the control is not operating
effectively, but in fact it is operating effectively, the audit is likely
to be faulty because of reduced substantive tests.
d. If sample results indicate that the control is not operating
effectively, but in fact it is operating effectively, control risk will be
assessed too low.
a. If sample results indicate that the control is operating
effectively, but in fact it is not, control risk will be assessed too
low.

For which of the following populations would probability-


proportional-to-size sampling be most likely to be the most
efficient technique in confirming accounts receivable?

a. 2,000 accounts with a standard deviation of $650, with few


misstatements expected.
b. 2,000 accounts with a standard deviation of $25, with few
misstatements expected.
c. 6,000 accounts with a standard deviation of $650, with many
misstatements expected.
d. 6,000 accounts with a standard deviation of $25 with many
misstatements expected.
a. 2,000 accounts with a standard deviation of $650, with few
misstatements expected.

The use of probability-proportional-to-size sampling would be


least likely to be efficient if:

a. Commercial receivable accounts are being audited.


b. Statistical inferences are to be made.
c. Many accounts are known to be in error.
d. The population has a high total dollar value.
c. Many accounts are known to be in error.

In the past, the auditors have found that the book value of a
receivable account has been related to the amount the account is
misstated (i.e., large accounts have large misstatements and
small accounts have small misstatements). Which of the following
techniques is most likely to be efficient?

a. Mean-per-unit estimation.
b. Ratio estimation.
c. Difference estimation.
d. Sequential sampling estimation.
b. Ratio estimation.

Which of the following statistical sampling plans does not use a


fixed sample size for tests of controls?

a. Dollar-unit sampling.
b. Sequential sampling.
c. Probability proportional to size (PPS) sampling.
d. Variables sampling.
b. Sequential sampling.

An advantage of statistical sampling over non-statistical sampling


is that statistical sampling helps an auditor to

a. Eliminate the risk of non-sampling errors.


b. Reduce the level of audit risk and materiality to a relatively low
amount.
c. Measure the sufficiency of evidential matter obtained.
d. Minimize the failure to detect errors and fraud.
c. Measure the sufficiency of evidential matter obtained.
Which of the following is an element of sampling risk?
a. choosing an audit procedure that's inconsistent w/ audit
objectives
b. choosing a sample size that's too small to achieve sampling
objective
c. failing to detect an error on previously inspected doc.
d. failing to perform audit proced. req'd by sampling plan
b. choosing a sample size that's too small to achieve sampling
objective

The primary reason for an auditor to use stat. samp. is to:


a. obtain smaller sample than req'd by nonstat samp
b. obtain sample more rep. of pop. than nonstat samp
c. allow the auditor to quantify and control the risk of making an
incorrect decision based on sample evid.
d. meet req. of prof. standards
c. allow the auditor to quantify and control the risk of making an
incorrect decision based on sample evid.

The statistical terms that roughly correspond to "quantities" and


"deviation rate" respectively are:
variables and attributes

A sampling method that can be used to est. the $ mis of an acct


bal but isn't based on normal curve math is:
a. discovery sampling
b. MPU sampling
c. attribute sampling
d. PPS Sampling
d. PPS Sampling

To test the AR file to compute an estimated $ total, the auditor


could use any one of the following sampling techniques except:
a. difference estimation
b. unstratified mean-per-unit est.
c. ratio est.
d. attribute est.
d. attribute est.

Each time an auditor draws a conclusion based on evidence


drawn from a sample, an additional risk, sampling risk, is
introduced. An example of sampling risk is:
a. projecting results beyond pop tested
b. using an improper audit procedure
c. incorrectly applying an audit procedure
d. drawing an erroneous conclusion from the sample data
d. drawing an erroneous conclusion from the sample data

In examining cash disbursements, an auditor plans to choose a


sample using systematic selection with a random start. The
primary advantage of such a systematic selection is that
population items
a. Which include errors will not be overlooked when the auditor
exercises compatible reciprocal options.
b. May occur in a systematic pattern, thus making the sample
more representative.
c. May occur more than once in a sample.
d. Do not have to be prenumbered in order for the auditor to use
the technique.
d. Do not have to be prenumbered in order for the auditor to use
the technique.

An auditor plans to examine a sample of 30 purchase orders for


proper approval as prescribed by the client's internal accounting
control procedures. One of the purchase orders in the chosen
sample of 30 cannot be found, and the auditor is unable to use
alternative procedures to test whether that purchase order was
properly approved. The auditor should
A) Choose another purchase order to replace the missing
purchase order in the sample.
B) Consider this compliance test invalid and proceed with
substantive tests because internal control cannot be relied upon.
C) Treat the missing purchase order as a deviation for the
purpose of evaluating the sample.
D) Select a completely new set of 30 purchase orders.
C) Treat the missing purchase order as a deviation for the
purpose of evaluating the sample.

An underlying feature of random-based selection of items is that


each

a. Stratum of the accounting population be given equal


representation in the sample.

b. Item in the accounting population be randomly ordered.

c. Item in the accounting population should have an opportunity to


be selected.

d. Item must be systematically selected using replacement.


c. Item in the accounting population should have an opportunity to
be selected.

An auditor initially planned to use unrestiricted random sampling


with replacement in the audit of accounts receivable. Later, the
auditor decided to use unrestriced random sampling without
replacement. As a result of this decision, the sample size should:
decrease

An auditor desires to use a table of a random digits to select a


sample from a population of documents which have the following
broken # sequences: 0001-1000, 2000-5000, and 8000-11000.
Which of the following is the most efficient approach to overcome
the problem of broken number sequences?
Deduct 4 digit constant values from the second and third
sequences, choose the appropriate random numbers, and add
the constants back to the individual numbers.

Which of the following is least desirable for use by an auditor?


a. systematic sampling
b. stratified sampling
c. block selection
d. sequential selection
c. block selection

The group of items about which the auditor wants to estimate


some characteristic is called the:
a. population
b. sample
c. sampling unit
a. population

Using random numbers to select a sample:


a. is required for a variables sampling plan
b. is likely to result in unbiasedness
c. results in a representative sample
d. allows auditors to use smaller samples
b. is likely to result in unbiasedness

In obtaining a sample for the purpose of reaching a conclusion


about the population from which it is drawn, the auditor:
can statistically quantify the sample results if and only if the
sample was selected in a way that each population item has an
equal or known probability of selection

Which of the following is correct concerning stat. sampling in tests


of controls?

a. the pop size has little or no effect on determine sample size


except for very small populations.
b. the expected deviation rate has little or no effect on determining
sample sizes except for very small populations
c. as the population size doubles, so should the sample size
d. for a tolerable rate, a larger sample size should be selected as
the expected population deviation rate decreases
a. the pop size has little or no effect on determine sample size
except for very small populations.

When assessing the tolerable deviation rate (TDR), the auditor


should consider that while deviations from control procedures
increase the risk of material misstatements, such deviations may
not necessarily result in errors. This explains why:
A) Deviations from examined control procedures at a given rate
would normally be expected to result in a higher rate of dollar
errors.
B) A recorded disbursement that is properly authorized may
nonetheless contain a material dollar error.
C) A recorded disbursement that is not properly authorized may
nonetheless be recorded properly in the cash disbursements
journal.
D) Deviations would result in dollar errors in the accounting
records only when they occurred in different transactions.
C) A recorded disbursement that is not properly authorized may
nonetheless be recorded properly in the cash disbursements
journal.

In the examination of the financial statements of Delta Company,


the auditor determines that in performing a test of internal control
effectiveness, the rate of error in the sample does not support the
auditor's preconceived notion of a tolerable occurrence rate when,
in fact, the actual error rate in the population does meet the
auditor's notion of effectiveness. This situation illustrates the risk
of

A. assessing CR too high


B. assessing CR too low
C. incorrect rejection
D. incorrect acceptance
A. assessing CR too high

Top of table: 1.) Deviation rate exceeds tolerable rate.


2.)Deviation rate is less than tolerable rate.
(true state of pop)
Side of table:3.) Deviation rate exceeds tolerable rate.
4.)Deviation rate is less than tolerable rate.
(auditor's est. based on sample results)

I- 1.) and 3.)


II- 2.) and 3)
III- 1.) and 4.)
IV- 2.) and 4.)

In which 2 situations would the auditor have properly assessed


CR?
I and IV

Top of table: 1.) Deviation rate exceeds tolerable rate.


2.)Deviation rate is less than tolerable rate.
(true state of pop)
Side of table:3.) Deviation rate exceeds tolerable rate.
4.)Deviation rate is less than tolerable rate.
(auditor's est. based on sample results)

I- 1.) and 3.)


II- 2.) and 3)
III- 1.) and 4.)
IV- 2.) and 4.)

The auditor assessed CR too high and thereby increases sub.


testing. Which situation?
II

The tolerable rate of deviations for a test of a control is generally:


A. Lower than the expected rate of deviations in the related
accounting records.

B. Higher than the expected rate of deviations in the related


accounting records.

C. Identical to the expected rate of deviations in the related


accounting records.

D. Unrelated to the expected rate of deviations in the related


accounting records.
B. Higher than the expected rate of deviations in the related
accounting records.

An auditor might use discovery sampling to:

a. compute the UPL of an infrequently occurring error


b. compute the UPL of a frequently occurring error
c. Estimate the $ value of a fraud
d. Determine how many invoices were paid twice
d. Determine how many invoices were paid twice

When planning an attribute sampling application, the difference


between the elected deviation rate and the maximum tolerable
deviation rate is the planned:

a. precision
b. reliability
c. dispersion
d. skewness
a. precision
An auditor is designing a sampling plan to test the accuracy of
daily production reports over the past 3 years. All of the reports
contain the same information except that Friday reports also
contain weekly totals and are prepared by managers rather than
by supervisors. Production normally peaks near the end of a
month. If the auditor wants to select two reports per month using
an interval sampling plan, which of the following techniques
reduces the likelihood of bias in the sample?

A. Estimating the rate of misstatements in the population.


B. Increasing the confidence level.
C. Using multiple random starts.
D. Increasing the precision
C. Using multiple random starts.

Which of the following factors are considered in determine the


sample size for a test of controls?
Expected Deviation Rate Tolerable Deviation Rate
A. Yes, Yes
B. No, No
C. No, Yes
D. Yes, No
A. Yes, Yes

In applying variables sampling, an auditor attempts to:


A. estimate a qualitative characteristic of interest
B. Determine various rates of occurrence for specified attributes
C. Discover at least 1 instance of a critical deviation.
D. Predict a monetary population value within a range of
precision.
D. Predict a monetary population value within a range of
precision.
In estimation sampling for variables, which of the following must
be known in order to estimate the appropriate sample size
required to meet the auditor's needs in a given situation?
A. qualitative aspects of mis.
B. total $ amount of pop.
C. # of mis. in pop.
D. estimated deviation rate in pop.
D. estimated deviation rate in pop.

An internal auditor is using variables estimation as the statistical


sampling technique to estimate the monetary value of a large
inventory of parts. Given a sample standard deviation of $400, a
sample size of 400, and a 95% two-tail confidence interval, what
precision can the auditor assign to the estimate of the mean dollar
value of a part?
a. +/- $39
b. +/- $2
c. +/- $52
d. +/- $20
a. +/- $39

A statistical sample from an inventory containing a total of 10,000


items produced a sample mean equal to $25, and a standard
error of the mean equal to $1.00. What is the interval estimate of
the total value of the inventory at the 95 percent confidence level
(assume UR = 2.0)?
a. $230,000 to $270,000.
b. $240,000 to $260,000.
c. $240,450 to $259,550.
d. $250,000 to $270,000.
a. $230,000 to $270,000.

In estimating the total value of supplies on repair trucks, Baker


Company draws random samples from two equal-sized strata of
trucks. The mean value of the inventory stored on the larger
trucks (stratum 1) was computed at $1,500, with a standard
deviation of $250. On the smaller trucks (stratum 2), the mean
value of inventory was computed as $500, with a standard
deviation of $45. If Baker had drawn an unstratified sample from
the entire population of trucks, the expected mean value of
inventory per truck would be $1,000, and the expected standard
deviation would be
A. $147.50
B. Greater than $250
C. Less than $45
D. Between $45 and $250 but not $147.50
B. Greater than $250

An auditor's finding was stated as follows: "Twenty of one


hundred randomly selected items tested revealed that $200 of
cash discounts on purchases were lost." This variables sampling
finding is deficient because the
a. Recommendation specifies no action
b. Sampling methodology is not defined
c. Amount is not material
d. Probable effect on the entire population is not provided
d. Probable effect on the entire population is not provided

In a sampling application, the standard deviation represents a


measure of the:
a. expected error rate
b. level of confidence desired
c. degree of data variability
d. extent of precision achieved
c. degree of data variability

In determining the sample size for variables sampling, the internal


auditor requires some knowledge of the variability of the
population. In obtaining this prelim. info., the internal auditor:
A. can seldom rely on PY sample results
B. frequently take a convenience pilot sample of 30-50 items and
uses this to est. the variability of the pop.
C. frequently take a convenience pilot sample of 30-50 items and
applies audit tests to these items to estimate the variability in the
pop. of audit values
D. frequently take a convenience pilot sample of 30-50 items and
compute the range in the sample to use the range as an estimate
of the variability of the pop.
B. frequently take a convenience pilot sample of 30-50 items and
uses this to est. the variability of the pop.

A construction company has an invntory of 1000 homes under


construction with a recorded BV=$22,222,222. The pop. standard
dev. was initially estimated to be $3000; desired precision and
confidence level were $247,500 and 90%. A sample of 400
homes was taken for use in mean-per-unit estimation. Audit
results revealed an estimated inventory value of $22M with a
precision of $291,176.

The sample standard deviation, which is different from that


originally estimated, affected which of the following?
A. est. inv. value of $22M
B. calculated precision of $291, 176
C. calculated sample size of 400
D. desired precision of $247,500
B. calculated precision of $291, 176

A construction company has an inventory of 1000 homes under


construction with a recorded BV=$22,222,222. The pop. standard
dev. was initially estimated to be $3000; desired precision and
confidence level were $247,500 and 90%. A sample of 400
homes was taken for use in mean-per-unit estimation. Audit
results revealed an estimated inventory value of $22M with a
precision of $291,176.
The finite pop. correction factor wasn't used to calculate the
required sample size. The application of the finite correction factor
to the precision interval will cause:
A. precision interval to become smaller
B. precision interval to become larger
C. reliability to decrease
D. reliability to increase
A. precision interval to become smaller

In an application of mean per unit sampling, the following


information has been obtained:
Reported book value $600,000
Point estimate (estimated total value) 591,000
Allowance for sampling risk (precision) +- 22,000
Tolerable error +- 45,000
The appropriate conclusion would be that the reported book value
is:
A. acceptable only if the risk of incorrect rejection is at least twice
the risk of incorrect acceptance
B. Acceptable
C. Not acceptable
D. Acceptable only if the risk of incorrect acceptance is at least
twice the risk of incorrect rejection
B. Acceptable

An auditor selects a preliminary sample of 100 items out of a


population of 1,000 items. The sample statistics generate an
arithmetic mean of $120, a standard deviation of $12, and a
standard error of the mean of $1.20. If the sample was adequate
for the auditor's purposes and the auditor's desired precision was
plus or minus $2,000, the minimum acceptable dollar value of the
population would be
A. $122K
B. $120K
C. $118K
D. $117.6K
C. $118K

In which of the following cases would the auditor be most likely to


conclude that all of the items in an account under consideration
should be examined rather than tested on a sample basis?
Measure of Tolerable Mis Is: Mis. Expected to be:

A. large, low
B. small, high
C. large, high
D. small, low
B. small, high

Which of the following models expresses the general relationship


of risks associated with the auditor's assessment of CR, AP, and
AR that would lead the auditor to conclude that additional
substantive tests of details of an account balance are not
necessary?

A. 20% AP, 40% CR, 10% AR


B. 20% AP, 60% CR, 5% AR
C. 10% AP, 70% CR, 4.5% AR
D. 30% AP, 40% CR, 5.5% AR
A. 20% AP, 40% CR, 10% AR

How would increases in tolerable mis. and assessed level of CR


affect the sample size in a substantive test of details?
Inc. in Tol. Mis. Inc. in Assessed CR
A. Increase samp size, increase samp size
B. Increase samp size, decrease samp size
C. Decrease samp size, increase samp size
D. Decrease samp size, decrease samp size
C. Decrease samp size, increase samp size
While performing a substantive TOD, the auditor determined the
sample results supported the conclusion that the recorded acct
bal. was mat. mis. It was, in fact, not mat. mis. This situation
illustrates the risk of:

A. Incorrect rejection
B. Incorrect acceptance
C. Assessing CR too low
D. Assessing CR too high
A. Incorrect rejection

A number of factors influence the sample size for a substantive


test of details of an account balance. All other factors being equal,
which of the following would lead to a larger sample size?
A. lower assessed CR
B. increased use of AP to obtain evid. about particular assertions
C. smaller expected frequency of deviations
D. smaller measure of tolerable misstatement
D. smaller measure of tolerable misstatement

Using the following results from a variable sample, compute the


standard error of the mean.
Population size=10,000
Sample size=144
Sample standard deviation=$24.00
Confidence level=90% (UR=1.65)
Mean=$84.00

A. $60
B. $7
C. $2.30
D. $2.00
D. $2.00

24/sqrt(144)=2
The auditors of Smith Electronics wish to limit the audit risk of
material misstatement in the test of accounts receivable to 5
percent. They believe that inherent risk is 100%, control risk is
50%, and there is a 40% risk that material misstatement could
have bypassed the client's system of internal control. What is the
maximum detection risk the auditors should specify in their
substantive procedures of details of accounts receivable?

A. 5%
B. 10%
C. 20%
D. 25%
D. 25%

0.05/(.5*.4)

An auditor randomly selects 100 items of finished goods perpetual


inventory, physically counts them, and computes an "audited
amount" for each (calculated as quantity times unit cost per
production reports). The internal auditor then compares the
audited amount with the "carrying amount" (inventory cost per
perpetual inventory records) and uses difference estimation to
estimate the correct total for the finished goods inventory. Results
of the 100-item sample: Total audited amount $605,000
Total carrying amount (of these 100 items) $630,000 Number of
items incorrectly stated (out of 100) 17
The total carrying amount of the entire finished goods inventory
(1,100 items) is $6,988,000. On the basis of difference estimation,
the auditor's best guess ("point estimate") as to the correct total is
A. $6,655,000
B. $6,713,000
C. $6,963,000
D. $7,263,000
B. $6,713,000
When an internal auditor uses the probability-proportional-to-size
sampling to examine the total value of invoices, each invoice:
A. has an equal probability of being selected
B. can be represented by no more than $1 unit
C. has an unknown probability of being selected
D. has a probability proportional to its dollar value of being
selected
D. has a probability proportional to its dollar value of being
selected

An auditor may use either of two statistical sampling approaches


in substantive testing, PPS and classical variables sampling. PPS
sampling is primarily acceptable in testing for:
A. # of mis. in YE sales cutoff
B. overstatement of AR
C. a credit balance in AR
D. proper segregation of duties in AR collections
B. overstatement of AR

Which of the following would be an improper technique when


using PPS sampling in an audit of AR?
A. combining negative and positive $ error item amounts in the
appraisal of a sample
B. using a sampling technique in which the same acct bal could
be selected more than once
C. selecting a random starting point and then sampling every
"nth" dollar unit (systematic sampling)
D. defining the sampling unit in the pop. as an individual dollar
and not as an individual acct bal.
A. combining negative and positive $ error item amounts in the
appraisal of a sample

In selecting a sample using PPS sampling, the dollar is the


sampling unit. Thus, if the 300th dollar of invoices is selected:
A. only that dollar is audited
B. Only an invoice with exactly $300 is audited
C. an invoice of less than $300 cannot be selected
D. the invoice containing the 300th dollar is audited
D. the invoice containing the 300th dollar is audited

An internal auditor has been assigned to take a PPS sample of a


population of vouchers in the purchasing department. The
population has a total BV of $300K. The internal auditor believes
that a max. mis. of $900 is acceptable and would like to have 95%
confidence in the results. (reliability factor=3)

voucher balance cumulative balance


1 150 100
2 150 250
3 40 290
4 200 490
5 10 500
6 290 790
7 50 840
8 190 1030
9 20 1050
10 180 1230

Given a random start of $50 as the first dollar amount, what is the
number of the 4th voucher to be selected, assuming the sample
size will be 1000?
A. 4
B. 6
C. 7
D. 8
D. 8

Which of the following statements is true concerning PPS


sampling, also known as dollar-unit sampling?
A. the sampling distribution should approximate the normal
distribution
B. overstated units have a lower probability of sample selection
that units that are understated
C. the auditor controls the risk of incorrect acceptance by
specifying that risk level for the sampling plan
D. the sampling interval is calculated by dividing the number of
physical units in the population by the sample size
C. the auditor controls the risk of incorrect acceptance by
specifying that risk level for the sampling plan

A sampling plan is needed to test for overstatement of a $3M


accounts payable book balance. The auditor determines that a
$100K mis. is material and 95% confidence interval is
appropriate. Sample size=90. The sampling plan most likely used
is:
A. stop and go
B. cluster sampling
C. PPS sampling
D. attribute sampling
C. PPS sampling

In a PPS sample with a sampling interval of $15K, an auditor


discovered that a sampled AR with a recorded amount of $20K
was overstated by $4K. The projected misstatement of this
sample was:
A. $3K
B. $3750
C. $4K
D. $12K
C. $4K

An auditor is considering the use of PPS sampling. This technique


is likely to be especially beneficial if:
A. the auditor is interested in testing the proper valuation AP
B. the auditor believes that the items to be tested are just as likely
to be overstated as understated
C. the auditor is interested in testing the accuracy and valuation of
AR
D. the error rate in the pop. is believed to be quite large
C. the auditor is interested in testing the accuracy and valuation of
AR

T/F: One of the basic features of statistical sampling is random


selection.
True

T/F: The frame may contain units that do not belong to the
population as long as it is possible to select the sample from units
that do belong.
True

T/F: Auditors typically use sampling without replacement.


True

T/F: In PPS selection, the cumulative dollar amount of population


items is used to select sample units, and all items in the
population must be positive amounts.
True

T/F: A valid statistical evaluation cannot be made of a biased


sample.
True

T/F: Using more than one random start in systematic selection is


a safeguard against bias caused by a sequence of items in the
population that corresponds to the sampling interval.
True
T/F: Computer generation is usually more efficient than manual
use of a random number table. There is no discard problem and
the numbers are in population sequence.
False

T/F: A nonstatistical sample must, according to SAS No. 39, be


selected using a method that can be expected to produce a
representative sample. A random-based selection method can be
expected to produce a representative sample.
True

T/F: Haphazard selection is acceptable only for a nonstatistical


sample.
True

T/F: If an auditor selects a sample randomly, he is using statistical


sampling.
False

T/F: Two broad classes of statistical sampling plans are attribute


sampling and discovery sampling.
False

T/F: SASs require the use of statistical sampling in selected


situations.
False

T/F: Nonstatistical sampling permits objective determination of


sample risk.
False

T/F: The failure to define the nature of an audit error is an


example of a sampling error.
False

T/F: Statistical sampling requires that selected audit judgments be


quantified but not eliminated
True

T/F: If an auditor examines 100% of the items in a population,


uncertainty is not eliminated
True

T/F: Statistical sampling requires that a sample be random


True

T/F: Valid sampling frames may contain units that don't belong to
the population as defined.
True

T/F: Sampling with replacement is more widely used by auditors


than sampling without replacement.
False

T/F: If a population is numbered from 6000-12000, a five-digit


number must be used to generate a sample from a random
number table
False

T/F: Negative amounts should be eliminated from a population


total before PPS sampling is applied
True

T/F: A biased sample cannot be evaluated statistically.


True

T/F: More than 1 random start should be used if systematic


selection is employed.
True

T/F: Random systemic selection is preferred relative to systematic


selection with one random start.
True
T/F: Computer generation is generally a less effective method
used to generate random numbers relative to a random-number
table
False

T/F: Random sampling techniques can be used for non statistical


sampling.
True

T/F: Haphazard selection can't be used in statistical sampling.


True

T/F: If the auditor expects 0 or near 0 deviations in a sample,


sequential sampling may be efficient
True

T/F: Performing a thorough qualitative analysis on each observed


deviation may be more informative than a quantitative projection
of a sample.
True

T/F: It's not necessary to perform qualitative analysis if the


achieved UPL is less than or equal to the tolerable rate
(acceptable UPL)
False

T/F: If the planned assessed level of CR is low, achieved UPL


generally shouldn't exceed 7%
True

T/F: The risk of placing reliance on IC when deviations are


excessive is referred to as the risk of assessing CR too high
False

T/F: If the tolerable rate (acceptable UPL)is less than the


achieved UPL in a sequential application it's generally not in
accordance with due professional care to increase the tolerable
rate
True

T/F: The minimum sample size for 95% reliability and 5%


tolerable rate for a fixed-sample-size attribute sample is 59.
True

T/F: The risk of assessing CR too low is generally set at 20% or


higher
False

T/F: To determine the accounts sample size for a discovery


sampling application, the approximate pop. size must be known
True

T/F: The auditor's assessed level of CR is usually based solely on


tests of controls that use audit sampling
False

T/F: The decision to use a fixed or a sequential sampling plan


depends on which plan the auditor believes will be most efficient
in the circumstances.
True

T/F: If an accountant wishes to use a table of random digits to


select random sample, he must first find a table that conforms to
the numbering employed by the items in the population the
accountant wishes to sample
False

T/F: If a usable number appears more than once in the table of


random digits during the selection of the sample, the item should
be included in the sample only once and another number should
be selected
True
T/F: A random sample of at least 50 items would have to be
discarded if it produced 1 item disproportionately large in relation
to the other items selected
False

T/F: The effect of the inclusion by chance of a very large or a very


small item in a random sample can be lessened by increasing the
size of the sample
True

T/F: The reliability specified by the accountant for a sample


estimate expresses the degree of confidence that the true value
will be within the computed precision interval
True

T/F: The standard deviation is a measure of variability of items in


the population
True

T/F: Variability of items in the population is a factor that usually


causes the point estimate of the population and its true value to
be different
True

T/F: It's necessary to determine the true standard deviation for a


population to determine the size of the sample to be drawn from
that population
False

T/F: The standard error of the mean generally will be less than the
estimated std dev computed on the basis of a sample
True

T/F: precision and reliability have no meaning unless paired with


each other
True
T/F: variable sampling is used primarily for TOC
False

T/F: The greater the tolerable mis. (amount considered material),


the smaller the sample size
True

T/F: 1-AR=confidence level


False

T/F: a graph of most accounting pop. generally will resemble a


normal distribution
False

T/F: audit hypothesis testing cannot be used unless the actual


value of the universe std dev is known
False

T/F: The assessed level of CR varies inversely with beta risk


False

T/F: a decision interval is constructed based on estimated audited


value +/1 precision
False

T/F: 1-reliability=alpha risk


True

T/F: if a client acct bal. is overstated by an amount less than tol.


mis., beta risk for the smaller amount is greater than beta risk for
the tol. mis.
False

T/F: Beta risk is usually measured in terms of a mat. mis.


True

T/F: A" is always smaller than A' and A


False

T/F: Any combination of CR and AP whose product equals 0.50


permits a tolerable beta risk of 0.10 for an audit risk of 0.05
True

T/F: Sampling units with 0 or negative book values must be


treated separately when PPS sampling is used
True

T/F: in a PPS substantive sampling application, the risk level


specified is beta risk
True

T/F: PPS sampling is a modified form of discovery sampling


False

T/F: a PPS sample may be selected via a random number table


(unrestricted random sampling) or systematic sampling
True

T/F: physical units (account items) that are overstated have a


lower probability of selection in a PPS sampling application
False

T/F: to use PPS sampling for testing controls, the auditor denies
attributes just as he or she would when using attribute sampling
True

T/F: PPS sampling is actually a form of stratified sampling


True

T/F: if a PPS sample indicates that the proportion of $


transactions associated with a control deviation is greater than
was planned, internal control can be assessed as planned
False
T/F: when systematic PPS sampling is used, all accounts
exceeding the total BV/sample size are examined on a 100%
basis
True

What is the primary objective of using stratification as a sampling


method in auditing?

a) to increase the risk of incorrect acceptance at which a decision


will be reached from the results o the sample selected
b) to determine the deviation rate for a given characteristic in the
population being studied
c) to decrease the effect of variance in the total population
d) to determine the precision interval of the sample selected
c

The risk of incorrect acceptance and the likelihood of assessing


control risk too low relate to the

a) effectiveness of the audit


b) efficiency of the audit
c) preliminary estimates of materiality levels
d) allowable risk of tolerable error
a

When performing tests of controls over cash disbursements, a


CPA may use a systematic sampling technique with a start at any
randomly selected item. The biggest disadvantage of this type of
sampling is that the items in the population
a) must be recorded in a systematic pattern before the sample
can be drawn
b) may occur in a systematic pattern, thus destroying the sample
randomness
c) may systematically occur more than once in the sample
d) must be systematically replaced in the population after
sampling
b

What is an auditor's evaluation of a statistical sample for


attributes when a test of 50 documents results in 3 deviations if
the tolerable rate is 7%, the expected population deviation rate is
5%, and the allowance for sampling risk is 2%?

a) Modify the planned assessed level of control risk because the


tolerable rate plus the allowance for sampling risk exceeds the
expected population deviation rate
b) accept the sample results as support for the planned assessed
level of control risk because the sample deviation rate plus the
allowance for sampling risk exceeds the tolerable rate
c) accept the sample results as support for the planned assessed
level of control risk because the tolerable rate less the allowance
for sampling risk equals the expected population deviation rate
d) modify the planned assessed level of control risk because the
sample deviation rate plus the allowance for sampling risk
exceeds the tolerable rate
d

If fraud or gross error is suspected in the population, the auditor


would most likely use

a) variables sampling
b) attribute sampling
c) discovery sampling
d) dollar-unit sampling
c

In a probability-proportional-to-size sample with a sampling


interval of $10,000, an auditor discovered that a selected account
receivable with a recorded amount of $5,000 had an audit amount
of $2,000. The projected error of this sample was

a) $3,000
b) $4,000
c) $6,000
d) $8,000
c

An auditor is performing substantive tests of pricing and


extensions of perpetual inventory balances consisting of a large
number of items. Past experience indicates numerous pricing and
extension errors. Which of the following statistical sampling
approaches is most appropriate?

a) unstratified mean-per-unit
b) probability-proportional-to-size
c) difference estimation
d) ratio estimation
c

What is the auditor's statistical conclusion when a sample of 50


items yields two exceptions if the tolerable exception rate is 5%,
the expected population exception rate is 1%, the ARACR is 10%
and the CUER is 10%?

a) accept the sample results since the sample error rate (4%) is
less than the TER (5%)
b) reject the sample results because the CUER (10%) exceeds
the TER (5%)
c) accept the sample results because the CUER (10%) does not
exceed the ARACR (10%)
d) reject the sample results because the sample error rate (4%)
exceeds the EPER (1%)
b

The 10,000 accounts receivable of DEF Company have a total


book value of $120,000. A CPA has selected and audited a
sample of 100 accounts with a total book value of $1,000 and an
audited value of $1,200.
- Using the difference estimation technique, estimated total
audited value of the population is:
A. $100,000.
B. $120,000.
C. $140,000.
D. $144,000
C. $140,000.

The 10,000 accounts receivable of DEF Company have a total


book value of $120,000. A CPA has selected and audited a
sample of 100 accounts with a total book value of $1,000 and an
audited value of $1,200.
- Using the ratio estimation technique, the estimated total audited
value of the population is:
A. $100,000.
B. $120,000.
C. $140,000.
D. $144,000
D. $144,000

The 10,000 accounts receivable of DEF Company have a total


book value of $120,000. A CPA has selected and audited a
sample of 100 accounts with a total book value of $1,000 and an
audited value of $1,200.
- Using the mean-per-unit estimation technique, the estimated
total audited value of the population is:
A. $100,000.
B. $120,000.
C. $140,000.
D. $144,000.
B. $120,000.

Using the mean-per-unit estimation method an auditor has


properly calculated the estimated total audited value of a
population as $200,000. Her sample included 200 of the
population's 40,000 items. She found that, in her sample the
average audited value was $1 less than the average book value.
What was the average audited value in the sample?
A. $4.
B. $5.
C. $6.
D. $10.
B. $5.

Using difference estimation, an auditor has taken a sample of 200


from a population's 40,000 items; that population has a book
value of $200,000. She found that in her sample the average
audited value was $4.20, while the average book value was
$5.20. What is the estimated total audited value of the
population?
A. $160,000.
B. $161,538.
C. $168,000.
D. $200,000.
A. $160,000
Using ratio estimation, an auditor has taken a sample of 200 from
a population's 40,000 items; that population has a book value of
$200,000. She found that in her sample the average audited
value was $4.20, while the average book value was $5.20. What
is the estimated total audited value of the population?
A. $160,000.
B. $161,538.
C. $168,000.
D. $200,000.
B. $161,538.

Using mean-per-unit estimation, an auditor has taken a sample of


200 from a population's 40,000 items; that population has a book
value of $200,000. She found that in her sample the average
audited value was $4.20, while the average book value was
$5.20. What is the estimated total audited value of the
population?
A. $160,000.
B. $161,538.
C. $168,000.
D. $200,000.
C. $168,000.

If the projected misstatement in a nonstatistical sampling is


$8,000, while the tolerable misstatement is $9,000, what would an
auditor likely conclude?
A. Since the projected misstatement is less than the tolerable
misstatement, the account is not misstated.
B. Since the projected misstatement is less than the tolerable
misstatement, the account is misstated.
C. The risk is high that the account is materially misstated.
D. The analysis has been improperly performed since the
projected misstatement is unequal to the tolerable misstatement.
C. The risk is high that the account is materially misstated.
A dual purpose test simultaneously.
A. Addresses two different accounts.
B. Functions as a substantive test and as a test of controls.
C. Functions as an analytical procedure and a substantive test.
D. Substantiates an ending balance and the transactions making
up the balance.
B. Functions as a substantive test and as a test of controls.

The 1000 accounts receivable of Winco Company have a total


book value of $20,000 (Average book value = $20). Bob Duffo,
CPA, has selected and audited a sample of 50 accounts with the
following mean values:
1. Book value of $19.
2. Audited value of $19.60.
- What is the estimated total audited value using mean per unit
sampling?
A. $19,000.
B. $19,600.
C. $20,000.
D. $20,632.
B. $19,600.

The 1000 accounts receivable of Winco Company have a total


book value of $20,000 (Average book value = $20). Bob Duffo,
CPA, has selected and audited a sample of 50 accounts with the
following mean values:
1. Book value of $19.
2. Audited value of $19.60.
- What is the estimated total audited value using difference
estimation sampling?
A. $19,387.
B. $19,400.
C. $19,600.
D. $20,600.
D. $20,600.
If all other factors specified in an attributes sampling plan remain
constant, decreasing the tolerable rate and decreasing the risk of
assessing control risk too low would have what effect on sample
size?
A. Increase.
B. Remain the same.
C. Decrease.
D. Indeterminate, depends upon exact change being made.
A. Increase.

If all other factors specified in an attributes sampling plan remain


constant, decreasing the tolerable rate and increasing the
estimated population deviation rate would have what effect on
sample size?
A. Increase.
B. Remain the same.
C. Decrease.
D. Indeterminate, depends upon exact change being made.
A. Increase.

An increase in the tolerable misstatement has what effect on the


planned allowance for sampling risk?
A. Increases.
B. Decreases.
C. No effect.
D. Indeterminate.
A. Increases.

Which of the following is not generally used for selecting


samples?
A. Random number tables.
B. Random number generators.
C. Physical representation numbers.
D. Systematic selection.
C. Physical representation numbers.
In performing a test of a control last year the auditors specified a
tolerable deviation rate of X percent. This year the auditors have
specified a tolerable rate of less than X percent. Assuming that all
other factors remain the same, which of the following is true
regarding the relationship between this year's sample size
compared to last year's sample size?
A. This year's sample is larger than last year's sample.
B. This year's sample is smaller than last year's sample.
C. This year's sample is equal to last year's sample.
D. This year's sample is indeterminate in relation to last year's
sample.
A. This year's sample is larger than last year's sample.

The auditor using nonstatistical attributes sampling, but who


nevertheless has chosen the sample in conformity with random
selection procedures:
A. Need not consider the risk of assessing control risk too low.
B. Has committed a nonsampling error.
C. Will have to use discovery sampling techniques to evaluate the
results.
D. Should compare the deviation rate of the sample to the
tolerable rate.
D. Should compare the deviation rate of the sample to the
tolerable rate.

When performing tests of controls over authorization of cash


receipts, which of the following sampling methods would be most
appropriate?
A. Attributes.
B. Ratio.
C. Stratified.
D. Variables.
A. Attributes.
Which of the following statistical sampling techniques involves
taking samples in a series of stages?
A. Systematic sampling.
B. Sequential sampling.
C. Continuous sampling.
D. Multiple location sampling.
B. Sequential sampling.

Which of the following is generally not true about statistical


sampling as compared to nonstatistical sampling?
A. Statistical samples are more representative of the population.
B. Statistical sample plans involve additional costs of evaluation.
C. Statistical sampling allows a more objective evaluation of
sample results.
D. Statistical sampling may assist the auditors in designing more
efficient samples.
A. Statistical samples are more representative of the population.

When using statistical sampling, which of the following need not


be known to evaluate the results of an attributes sample?
A. Sample size.
B. Risk of assessing control risk too low.
C. Number of deviations in the population.
D. Number of deviations found in the sample.
C. Number of deviations in the population.

When the auditors have decided to use statistical rather than


nonstatistical sampling, a disadvantage is that:
A. Designing efficient samples is more difficult.
B. The costs of training staff may be higher.
C. Sampling without replacement must be used.
D. Objectively evaluating results is impossible.
B. The costs of training staff may be higher.
Changing from a sampling plan using random selection with
replacement to random selection without replacement has what
effect on the required sample size?
A. Increases.
B. Decreases
C. No effect.
D. An indeterminate effect.
B. Decreases

When the auditors have chosen to test a control, what relationship


will the tolerable rate normally have when compared to the
expected rate of deviations in the sample?
A. Exceed.
B. Equal.
C. Be less than.
D. Indefinite.
A. Exceed.

Increases in the planned allowance for sampling risk have what


effect on required sample size?
A. Increases.
B. Decreases.
C. No effect.
D. Indeterminate.
B. Decreases.

When using sampling for tests of controls, which of the following


audit consequences may follow?
A. If sample results indicate that the control is operating
effectively, but in fact it is not, control risk will be assessed too
low.
B. If sample results indicate that the control is operating
effectively, but in fact it is not, control risk will be assessed too
high.
C. If sample results indicate that the control is not operating
effectively, but in fact it is operating effectively, the audit is likely
to be faulty because of reduced substantive tests.
D. If sample results indicate that the control is not operating
effectively, but in fact it is operating effectively, control risk will be
assessed too low.
A. If sample results indicate that the control is operating
effectively, but in fact it is not, control risk will be assessed too
low.

The auditors expect a population deviation rate of billing errors of


two percent, and have established a tolerable rate of five percent.
The sampling approach most likely to be used is:
A. Attributes sampling.
B. Stratified sequential sampling.
C. Discovery sampling.
D. None, as sampling does not seem appropriate in this situation.
A. Attributes sampling.

The auditors expect a population deviation rate of billing errors of


eight percent, and have established a tolerable rate of five
percent. The sampling approach most likely to be used is:
A. Attributes sampling.
B. Stratified random sampling.
C. Variables sampling.
D. None, as sampling does not seem appropriate in this situation.
D. None, as sampling does not seem appropriate in this situation.

Which of the following may be used to reduce the risk of


nonsampling errors?
A. Increasing the size of audit samples.
B. Stratifying audit samples.
C. Adequately planning audit samples.
D. Using statistical sampling techniques.
C. Adequately planning audit samples.
Which of the following types of risk is of critical importance to
auditors in performing tests of controls?
A. The risk of assessing control risk too low.
B. The risk of assessing control risk too high.
C. The risk of incorrect acceptance.
D. The risk of incorrect rejection.
A. The risk of assessing control risk too low.

The auditors' failure to recognize a misstatement in an amount or


a deviation in an internal control data processing procedure is
described as a:
A. Statistical error.
B. Sampling misstatement.
C. Standard error of the mean.
D. Nonsampling error.
D. Nonsampling error.

What effect does obtaining a sample with a standard deviation


larger than that estimated during planning have on the adjusted
allowance for sampling risk?
A. Increases.
B. Decreases.
C. No effect.
D. Indeterminate.
B. Decreases.

The 2000 accounts receivable of ABC Company have a total book


value of $40,000. Bob Rotter, CPA, has selected and audited a
sample of 100 accounts with a total book value of $1,950. The
audited value of the 100 accounts in the sample is $1,875.
-Using the mean-per-unit estimation technique, Rotter's estimate
(to the nearest dollar) of year-end accounts receivable balance
would be:
A. $37,500
B. $38,462
C. $38,500
D. $41,600
A. $37,500

The 2000 accounts receivable of ABC Company have a total book


value of $40,000. Bob Rotter, CPA, has selected and audited a
sample of 100 accounts with a total book value of $1,950. The
audited value of the 100 accounts in the sample is $1,875.
- Using the ratio estimation technique, Rotter's estimate (to the
nearest dollar) of year-end accounts receivable balance would be:
A. $37,500
B. $38,462
C. $38,500
D. $41,600
B. $38,462

The 2000 accounts receivable of ABC Company have a total book


value of $40,000. Bob Rotter, CPA, has selected and audited a
sample of 100 accounts with a total book value of $1,950. The
audited value of the 100 accounts in the sample is $1,875.
- Using the difference estimation technique, Rotter's estimate (to
the nearest dollar) of year-end accounts receivable balance would
be:
A. $37,500
B. $38,462
C. $38,500
D. $41,600
C. $38,500

The 3000 accounts receivable of DEF Company have a total book


value of $60,000. Bob Smith, CPA, has selected and audited a
sample of 100 accounts with a total book value of $2,100. Using
the difference estimation technique, Smith has properly estimated
a projected misstatement of an overstatement of $6,000 for the
entire population. The audited value of Smith's sample is:
A. $1,700
B. $1,800
C. $1,900
D. $2,300
C. $1,900

The 3000 accounts receivable of DEF Company have a total book


value of $60,000. Bob Smith, CPA, has selected and audited a
sample of 100 accounts with a total book value of $2,100. Using
the difference estimation technique, Smith has properly estimated
a projected misstatement of a $9,000 overstatement for the entire
population. The estimated total audited value of the population is:
A. $51,000
B. $58,000
C. $60,000
D. $69,000
A. $51,000

The 4000 accounts receivable of GHI Company have a total book


value of $60,000. Bob Smith, CPA, has selected and audited a
sample of 100 accounts with a total book value of $1,600. Using
the mean-per-unit estimation technique, Smith has properly
estimated a projected misstatement of a $8,000 overstatement for
the entire population. The audited value of Smith's sample is:
A. $1,300
B. $1,400
C. $1,600
D. $1,800
A. $1,300

Which of the following is a correct statement with respect to


evaluating results when using nonstatistical sampling for
substantive tests?
A. When the projected misstatement exceeds the tolerable
misstatement, the auditor should conclude that the population is
not misstated.
B. The closer the projected misstatement is to the tolerable
misstatement, the higher the risk of material misstatement.
C. When the projected misstatement is equal to zero, the auditors
may conclude with certainty that no misstatements exist in the
account.
D. When the projected misstatement percentage exceeds the risk
of incorrect acceptance the auditors will generally conclude that
the population is materially misstated.
B. The closer the projected misstatement is to the tolerable
misstatement, the higher the risk of material misstatement.

Assume a mean-per-unit estimation variables sampling


application with a tolerable misstatement of $70,000 and a book
value of $700,000. After performing the sampling plan, the
auditors calculated an adjusted allowance for sampling risk of
$45,000 and a point estimate of the population's total audited
value to be $650,000. Based on these results, the auditor would:
A. Because the tolerable misstatement exceeds the adjusted
allowance for sampling risk, conclude that the population does not
contain a material misstatement.
B. Because the total audited value tolerable misstatement
includes the book value, conclude that the population does not
contain a material misstatement.
C. Because the tolerable misstatement exceeds the adjusted
allowance for sampling risk, conclude that there is too great a risk
that the account balance is materially misstated.
D. Because the total audited value adjusted allowance for
sampling risk does not include the book value, conclude that there
is too great a risk that the account balance is materially misstated.
D. Because the total audited value adjusted allowance for
sampling risk does not include the book value, conclude that there
is too great a risk that the account balance is materially misstated.
Assume a mean-per-unit estimation variables sampling
application with a tolerable misstatement of $70,000 and a book
value of $700,000. After performing the sampling plan, the
auditors calculated an adjusted allowance for sampling risk of
$45,000 and a point estimate of the population's total audited
value to be $640,000. The projected misstatement based on this
sample is:
A. $ 5,000
B. $45,000
C. $60,000
D. $70,000
C. $60,000

The auditors have audited a sample with a standard deviation of


audited values larger than they had originally estimated. In this
situation, to maintain the risk of incorrect acceptance at its
predetermined level without increasing the size of the sample,
which of the following statements is correct?
A. The adjusted allowance for sampling risk will be smaller than
had been planned.
B. The adjusted allowance for sampling risk will be larger than
had been planned.
C. The risk of incorrect rejection will necessarily decrease.
D. The size of the population must be decreased.
A. The adjusted allowance for sampling risk will be smaller than
had been planned.

In the past, the auditors have found that the book value of a
receivable account has been related to the amount the account is
misstated (i.e., large accounts have large misstatements and
small accounts have small misstatements). Which of the following
techniques is most likely to be efficient?
A. Mean-per-unit estimation.
B. Ratio estimation.
C. Difference estimation.
D. Sequential sampling estimation.
B. Ratio estimation.

Which of the following statements is correct concerning the use of


nonstatistical sampling for substantive tests?
A. Its use is generally acceptable only for populations with an
immaterial book value.
B. It requires the use of structured sample size selection
techniques to be acceptable.
C. It may be especially useful in circumstances in which the
combination of inherent and control risk is at the maximum level.
D. Results will be projected to the population.
D. Results will be projected to the population.

Which of the following situations will result in the auditors


concluding that the risk of material misstatement is too high when
using nonstatistical sampling for substantive tests?
A. The projected misstatement exceeds the tolerable
misstatement.
B. The allowance for sampling risk exceeds the projected
misstatement.
C. The risk of incorrect acceptance exceeds the risk of incorrect
rejection.
D. The tolerable misstatement exceeds the sample net
misstatement.
A. The projected misstatement exceeds the tolerable
misstatement.

During the final planning of the accounts receivable program a


CPA specified a tolerable misstatement of $30,000, instead of the
$20,000 contained in the preliminary audit program. What would
be the impact of this change?
A. A decrease in population standard deviation.
B. An increase in sample standard deviation.
C. An increase in required sample size.
D. A decrease in the required sample size.
D. A decrease in the required sample size.

Which of the following is an advantage of systematic selection


over random number selection?
A. It provides a stronger basis for statistical conclusions.
B. It enables the auditor to use the more efficient "sampling with
replacement" tables.
C. There may be correlation between the location of items in the
population, the feature of sampling interest, and the sampling
interval.
D. It does not require establishment of correspondence between
random numbers and items in the population.
D. It does not require establishment of correspondence between
random numbers and items in the population.

Statistical sampling generally may be applied to test internal


control when the client's internal control procedures:
A. Depend primarily on appropriate segregation of duties.
B. Are carefully reduced to writing and are included in client
accounting manuals.
C. Leave an audit trail in the form of evidence of compliance.
D. Enable the detection of material fraud in the accounting
records.
C. Leave an audit trail in the form of evidence of compliance.

Which of the following statements is correct about the sample size


in statistical sampling when testing internal controls?
A. The auditor should consider the tolerable rate of deviation from
the controls being tested in determining sample size.
B. As the likely rate of deviation decreases, the auditor should
increase the planned sample size.
C. The allowable risk of assessing control risk too low has no
effect on the planned sample size.
D. Of all the factors to be considered, the population size has the
greatest effect on the sample size.
A. The auditor should consider the tolerable rate of deviation from
the controls being tested in determining sample size.

To determine sample size in an attribute sampling application,


what must be specified?
A. Population mean, expected error rate, allowance for sampling
risk.
B. Allowance for sampling risk, risk of assessing control risk too
low, standard deviation.
C. Allowance for sampling risk, risk of assessing control risk too
low, expected deviation rate.
D. Population mean, standard deviation, allowance for sampling
risk.
C. Allowance for sampling risk, risk of assessing control risk too
low, expected deviation rate.

The tolerable deviation rate in sampling for tests of controls is:


A. Used to determine the probability of the auditor's conclusion
based upon reliance factors.
B. The probability that the financial statements are not materially
in error.
C. A measure of the reliability of substantive tests.
D. The rate the auditor will tolerate without modifying the planned
assessment of control risk.
D. The rate the auditor will tolerate without modifying the planned
assessment of control risk.

In testing accounts receivable, an auditor sends out positive


confirmation requests to 100 randomly selected customers. A
customer returns the confirmation indicating that the balance is
correct when, in fact, the balance is overstated. This is an
example of:
A. Projected misstatement.
B. Sampling error.
C. Standard error.
D. Nonsampling error.
D. Nonsampling error.

An auditor wishes to estimate inventory shrinkage by weighing a


sample of inventory items. From experience, the auditor knows
that a few specific items are subject to unusually large amounts of
shrinkage. In using statistical sampling, the auditor's best course
of action is to:
A. Eliminate any of the items known to be subject to unusually
large amounts of shrinkage.
B. Increase the sample size to lessen the effect of the items
subject to unusually large amounts of shrinkage.
C. Stratify the inventory population so that items subject to
unusually large amounts of shrinkage are reviewed separately.
D. Continue to draw new samples until a sample is drawn which
includes none of the items known to be subject to large amounts
of shrinkage.
C. Stratify the inventory population so that items subject to
unusually large amounts of shrinkage are reviewed separately.

If all other factors specified in an attributes sampling plan remain


constant, changing the specified tolerable rate from 6% to 10%,
and changing the specified risk of assessing control risk too low
from 3% to 7% would cause the required sample size to:
A. Increase.
B. Remain the same.
C. Decrease.
D. Change by 4%.
C. Decrease.

Which of the following factors does an auditor need to consider in


planning a particular audit sample for a test of control?
A. Number of items in the population.
B. Total dollar amount of the items to be sampled.
C. Acceptable level of risk of assessing control risk too low.
D. Tolerable misstatement.
C. Acceptable level of risk of assessing control risk too low.

When using a statistical sampling plan, the auditors would


probably require a smaller sample if the:
A. Population increases.
B. Desired allowance for sampling risk decreases.
C. Desired risk of incorrect acceptance increases.
D. Expected deviation rate increases.
C. Desired risk of incorrect acceptance increases.

Which of the following statements is correct concerning statistical


sampling in tests of controls?
A. The population size has little effect on determining sample size
except for very small populations.
B. The expected population deviation rate has little or no effect on
determining sample size except for very small populations.
C. As the population size doubles, the sample size also should
double.
D. For a given tolerable rate, a larger sample size should be
selected as the expected population deviation rate decreases.
A. The population size has little effect on determining sample size
except for very small populations.

An advantage of using statistical sampling techniques is that such


techniques:
A. Mathematically measure risk.
B. Eliminate the need for judgmental decisions.
C. Define the values of allowance for sampling risk and tolerable
misstatement required to provide audit satisfaction.
D. Have been established in the courts to be superior to
judgmental sampling.
A. Mathematically measure risk.
In assessing sampling risk, the risk of incorrect rejection and the
risk of assessing control risk too high relate to the:
A. Efficiency of the audit.
B. Effectiveness of the audit.
C. Selection of the sample.
D. Audit quality controls.
A. Efficiency of the audit.

When performing a test of a control over cash disbursements, a


CPA may use a systematic sampling technique with a start at any
randomly selected item. The biggest disadvantage of this type of
sampling is that the items in the population:
A. Must be recorded in a systematic pattern before the sample
can be drawn.
B. May occur in a systematic pattern and destroy the sample
randomness.
C. May systematically occur more than once in the sample.
D. Must be systematically replaced in the population after
sampling.
B. May occur in a systematic pattern and destroy the sample
randomness.

While performing a substantive test of details during an audit, the


auditor determined that the sample results supported the
conclusion that the recorded account balance was materially
misstated. It was, in fact, not materially misstated. This situation
illustrates the risk of:
A. Incorrect acceptance.
B. Incorrect rejection.
C. Assessing control risk too high.
D. Assessing control risk too low.
B. Incorrect rejection.

An auditor plans to examine a sample of 20 checks for counter


signatures as prescribed by the client's control procedures. One
of the checks in the chosen sample of 20 cannot be found. The
auditor should consider the reasons for this limitation and:
A. Evaluate the results as if the sample size had been 19.
B. Treat the missing check as a deviation for the purpose of
evaluating the sample.
C. Treat the missing check in the same manner as the majority of
the other 19 checks, i.e., countersigned or not.
D. Choose another check to replace the missing check in the
sample.
B. Treat the missing check as a deviation for the purpose of
evaluating the sample

There are many kinds of statistical estimates that an auditor may


find useful, but basically every statistical estimate in auditing is of
either a quantity or of an error rate. The statistical terms that
roughly correspond to "quantities" and "occurrence rate",
respectively, are:
A. Attributes and variable.
B. Variables and attributes.
C. Constants and attributes.
D. Constants and variables.
B. Variables and attributes.

Which of the following best illustrates the concept of sampling


risk?
A. A randomly chosen sample may not be representative of the
population as a whole on the characteristic of interest.
B. An auditor may select audit procedures that are not appropriate
to achieve the specific objective.
C. An auditor may fail to recognize deviations in the documents
examined for the chosen sample.
D. The documents related to the chosen sample may not be
available for inspection.
A. A randomly chosen sample may not be representative of the
population as a whole on the characteristic of interest.
Which of the following statistical selection techniques is least
desirable for use by an auditor?
A. Systematic selection.
B. Stratified selection.
C. Block selection.
D. Sequential selection.
C. Block selection.

Various factors influence the sample size for a substantive test of


details of an account balance. All other factors being equal, which
of the following would lead to a larger sample size?
A. Lower assessment of control risk.
B. Greater reliance on analytical procedures.
C. Smaller expected frequency of misstatements.
D. Smaller measure of tolerable misstatement.
D. Smaller measure of tolerable misstatement.

An important statistic to consider when using a statistical


sampling audit plan is the population variability. The population
variability is measured by the:
A. Sample mean.
B. Standard deviation.
C. Standard error of the sample mean.
D. Estimated population total minus the actual population total.
B. Standard deviation.

In variables estimation sampling, the sample standard deviation is


used to calculate the
A. Point estimate of central tendency.
B. Tainting of the sample interval.
C. Risk of incorrect acceptance.
D. Adjusted allowance for sampling risk.
D. Adjusted allowance for sampling risk.
Use of the ratio estimation sampling technique to estimate dollar
amounts is inappropriate when:
A. The total book value is known and corresponds to the sum of
all the individual book values.
B. A book value for each sample item is unknown.
C. There are some observed differences between audited values
and book values.
D. The audited values are nearly proportional to the book value.
B. A book value for each sample item is unknown

Which of the following best illustrates the concept of sampling


risk?
A) An auditor may select audit procedures that are not
appropriate to achieve the specific objective.
B) A randomly chosen sample may not be representative of the
population as a whole on the characteristic of interest.
C) An auditor may fail to recognize errors in the documents
examined for the chosen sample.
D) The documents related to the chosen sample may not be
available for inspection.
B

The likelihood of assessing control risk too high is the risk that the
sample selected to test controls
A) Does not support the auditor's planned assessed level of
control risk when the true operating effectiveness of the control
justifies such an assessment.
B) Does support the auditor's planned assessed level of control
risk when the true operating effectiveness of the control does not
justify such an assessment.
C) Contains misstatements that could be material to the financial
statements when aggregated with misstatements in other account
balances or transaction classes.
D) Contains proportionately fewer monetary errors or deviations
from prescribed internal controls than exist in the balance or class
as a whole.
A

Sample size varies indirectly with which of the following:


A) Desired level of confidence.
B) Expected population deviation rate.
C) Tolerable deviation rate.
D) All of the above.
C

For which of the following audit tests would an auditor most likely
use attribute sampling?
A) Observation of employees who control mailroom receipts.
B) Examining supporting documentation for purchases for
evidence of proper authorization.
C) Examining invoices in support of the valuation of equipment
additions.
D) Selected accounts receivable for confirmation of account
balances.
B

An auditor plans to examine a sample of 40 accounts payable


invoices for proper approval as prescribed by the entity's internal
accounting control procedures. One of the invoices in the chosen
sample cannot be found, and the auditor is unable to use
alternative procedures to determine whether the invoice was
properly approved. The auditor should
A) Choose another invoice to replace the missing one in the
sample.
B) Consider this compliance test invalid and proceed with
substantive tests because internal control cannot be relied upon.
C) Treat the missing invoice as a deviation for the purpose of
evaluating the sample.
D) Select a completely new random set of 40 invoices.
C

Which of the following statements is true in an attribute sampling


plan where the tolerable deviation rate is 7%, the computed upper
deviation rate is 6.5%, the sample deviation rate is 2%, and the
risk of assessing control risk too low is 5%?
A) The auditor is likely to increase control risk because the
computed upper deviation rate is less than the risk of assessing
control risk too low.
B) The auditor is likely to decrease control risk because the
computed upper deviation rate is less than the risk of assessing
control risk too low.
C) The auditor is likely to determine that the results do not support
reliance on the control because the computed upper deviation
rate plus the sample deviation rate is greater than the tolerable
deviation rate.
D) The auditor is likely to determine that the results do support
reliance on the control because the computed upper deviation
rate is less than the tolerable deviation rate.
D

As a result of sampling procedures applied as tests of controls, an


auditor incorrectly assesses control risk (CR) lower than
appropriate. The most likely explanation for this situation is that
A) The deviation/failure rates of both the auditor's sample and the
population exceed the tolerable deviation rate [TDR].
B) The deviation/failure rate in the auditor's sample is less than
the TDR, but the deviation/failure rate in the population exceeds
the TDR.
C) The deviation/failure rate in the auditor's sample exceeds the
TDR, but the deviation/failure rate in the population is less than
the TDR.
D) The deviation/failure rates of both the auditor's sample and the
population are less than the TDR.
B

Which of the following best illustrates the components that make


up the upper deviation rate [UDR]?
A) Sample deviation rate + allowance for sampling risk.
B) Sample deviation rate + risk of assessing control risk too high.
C) Tolerable deviation rate + allowance for sampling risk.
D) Expected population deviation rate + allowance for sampling
risk.
A

When assessing the tolerable deviation rate (TDR), the auditor


should consider that while deviations from control procedures
increase the risk of material misstatements, such deviations may
not necessarily result in errors. This explains why
A) Deviations from examined control procedures at a given rate
would normally be expected to result in a higher rate of dollar
errors.
B) A recorded disbursement that is properly authorized may
nonetheless contain a material dollar error.
C) A recorded disbursement that is not properly authorized may
nonetheless be recorded properly in the cash disbursements
journal.
D) Deviations would result in dollar errors in the accounting
records only when they occurred in different transactions.
C

Which of the following statements is true regarding nonstatistical


sampling?
A) It quantifies the auditor's exposure to sampling risk.
B) Its use is required by the Public Company Accounting
Oversight Board for small public company audits.
C) Sample sizes for non-statistical sampling should be
comparable to statistical sampling.
D) It gives greater assurance than statistical sampling that
samples are randomly selected.
C

Which of the following types of evidence is most likely to utilize


sampling?
A) Scanning.
B) Confirmation.
C) Observation.
D) Analytical procedures.
B

Which of the following is true regarding technological advances


and auditing?
A) A well-controlled, automated accounting system that processes
routine transactions with few errors can reduce the number of
times auditors need to utilize sampling techniques in an audit.
B) A well-controlled, automated accounting system that processes
routine transactions with few errors can eliminate the number of
times auditors need to utilize sampling techniques in an audit.
C) Powerful audit software (e.g., ACL) has eliminated the need for
sampling in an audit.
D) Technology, though not there yet, will eventually eliminate the
need for auditors to rely on sampling in an audit.
A

The risk of incorrect rejection is associated with


A) Inherent risk.
B) Engagement risk.
C) Sampling risk.
D) Audit risk.
C

Which of the following types of statistical testing is likely to be


used for a test of controls?
A) Monetary-unit sampling.
B) Probability-proportional-to-size sampling.
C) Attribute sampling.
D) Classical variables sampling.
C

Most of the steps for planning and carrying out a nonstatistical


test of controls are the same as those for a statistical test of
controls. Which of the following represent the steps that could
differ between nonstatistical and statistical sampling as discussed
in the text?
A) Defining the control deviation conditions, determining the
sample size, calculating the upper deviation rate.
B) Defining the control deviation conditions, defining the sampling
unit, calculating the computed upper deviation rate.
C) Determining the sample size, selecting the sample items,
calculating the computed upper deviation rate.
D) Defining the sampling unit, selecting the sample items,
performing the audit procedures.
C

S-ar putea să vă placă și